11.07.2015 Views

The properties of force systems must be thoroughly understood by ...

The properties of force systems must be thoroughly understood by ...

The properties of force systems must be thoroughly understood by ...

SHOW MORE
SHOW LESS
  • No tags were found...

Create successful ePaper yourself

Turn your PDF publications into a flip-book with our unique Google optimized e-Paper software.

c02.qxd 10/29/07 1:38 PM Page 2626 Chapter 2 Force SystemsF– FVector ComponentsIn addition to combining <strong>force</strong>s to obtain their resultant, we <strong>of</strong>tenneed to replace a <strong>force</strong> <strong>by</strong> its vector components in directions which areconvenient for a given application. <strong>The</strong> vector sum <strong>of</strong> the components<strong>must</strong> equal the original vector. Thus, the <strong>force</strong> R in Fig. 2/3a may <strong>be</strong> replaced<strong>by</strong>, or resolved into, two vector components F 1 and F 2 with thespecified directions <strong>by</strong> completing the parallelogram as shown to obtainthe magnitudes <strong>of</strong> F 1 and F 2 .<strong>The</strong> relationship <strong>be</strong>tween a <strong>force</strong> and its vector components alonggiven axes <strong>must</strong> not <strong>be</strong> confused with the relationship <strong>be</strong>tween a <strong>force</strong>and its perpendicular* projections onto the same axes. Figure 2/3eshows the perpendicular projections F a and F b <strong>of</strong> the given <strong>force</strong> R ontoaxes a and b, which are parallel to the vector components F 1 and F 2 <strong>of</strong>Fig. 2/3a. Figure 2/3e shows that the components <strong>of</strong> a vector are not necessarilyequal to the projections <strong>of</strong> the vector onto the same axes. Furthermore,the vector sum <strong>of</strong> the projections F a and F b is not the vectorR, <strong>be</strong>cause the parallelogram law <strong>of</strong> vector addition <strong>must</strong> <strong>be</strong> used t<strong>of</strong>orm the sum. <strong>The</strong> components and projections <strong>of</strong> R are equal onlywhen the axes a and b are perpendicular.F 2F 1 R1R 2R 1R 2RFigure 2/4A Special Case <strong>of</strong> Vector AdditionTo obtain the resultant when the two <strong>force</strong>s F 1 and F 2 are parallelas in Fig. 2/4, we use a special case <strong>of</strong> addition. <strong>The</strong> two vectors are combined<strong>by</strong> first adding two equal, opposite, and collinear <strong>force</strong>s F and F<strong>of</strong> convenient magnitude, which taken together produce no external effecton the body. Adding F 1 and F to produce R 1 , and combining withthe sum R 2 <strong>of</strong> F 2 and F yield the resultant R, which is correct in magnitude,direction, and line <strong>of</strong> action. This procedure is also useful forgraphically combining two <strong>force</strong>s which have a remote and inconvenientpoint <strong>of</strong> concurrency <strong>be</strong>cause they are almost parallel.It is usually helpful to master the analysis <strong>of</strong> <strong>force</strong> <strong>systems</strong> in twodimensions <strong>be</strong>fore undertaking three-dimensional analysis. Thus the remainder<strong>of</strong> Chapter 2 is subdivided into these two categories.SECTION ATWO-DIMENSIONAL FORCE SYSTEMSjF yyθF xFix2/3 RECTANGULAR C OMPONENTS<strong>The</strong> most common two-dimensional resolution <strong>of</strong> a <strong>force</strong> vector isinto rectangular components. It follows from the parallelogram rulethat the vector F <strong>of</strong> Fig. 2/5 may <strong>be</strong> written asF F x F y(2/1)where F x and F y are vector components <strong>of</strong> F in the x- and y-directions.Each <strong>of</strong> the two vector components may <strong>be</strong> written as a scalar times theFigure 2/5*Perpendicular projections are also called orthogonal projections.


c02.qxd 10/29/07 1:38 PM Page 27Article 2/3 Rectangular Components 27appropriate unit vector. In terms <strong>of</strong> the unit vectors i and j <strong>of</strong> Fig. 2/5,F x F x i and F y F y j, and thus we may writeF F x i F y j(2/2)where the scalars F x and F y are the x and y scalar components <strong>of</strong> the vectorF.<strong>The</strong> scalar components can <strong>be</strong> positive or negative, depending onthe quadrant into which F points. For the <strong>force</strong> vector <strong>of</strong> Fig. 2/5, the xand y scalar components are both positive and are related to the magnitudeand direction <strong>of</strong> F <strong>by</strong>2 2F x F cos F F x F yFyβxF y F sin tan 1 F yF x(2/3)F x = F sin βF y = F cos βConventions for Describing Vector ComponentsWe express the magnitude <strong>of</strong> a vector with lightface italic type inprint; that is, F is indicated <strong>by</strong> F, a quantity which is always nonnegative.However, the scalar components, also denoted <strong>by</strong> lightface italictype, will include sign information. See Sample Problems 2/1 and 2/3 fornumerical examples which involve both positive and negative scalarcomponents.When both a <strong>force</strong> and its vector components appear in a diagram, itis desirable to show the vector components <strong>of</strong> the <strong>force</strong> with dashedlines, as in Fig. 2/5, and show the <strong>force</strong> with a solid line, or vice versa.With either <strong>of</strong> these conventions it will always <strong>be</strong> clear that a <strong>force</strong> andits components are <strong>be</strong>ing represented, and not three separate <strong>force</strong>s, aswould <strong>be</strong> implied <strong>by</strong> three solid-line vectors.Actual problems do not come with reference axes, so their assignmentis a matter <strong>of</strong> arbitrary convenience, and the choice is frequentlyup to the student. <strong>The</strong> logical choice is usually indicated <strong>by</strong> the way inwhich the geometry <strong>of</strong> the problem is specified. When the principal dimensions<strong>of</strong> a body are given in the horizontal and vertical directions, forexample, you would typically assign reference axes in these directions.Determining the Components <strong>of</strong> a ForceDimensions are not always given in horizontal and vertical directions,angles need not <strong>be</strong> measured counterclockwise from the x-axis,and the origin <strong>of</strong> coordinates need not <strong>be</strong> on the line <strong>of</strong> action <strong>of</strong> a <strong>force</strong>.<strong>The</strong>refore, it is essential that we <strong>be</strong> able to determine the correct components<strong>of</strong> a <strong>force</strong> no matter how the axes are oriented or how the anglesare measured. Figure 2/6 suggests a few typical examples <strong>of</strong> vectorresolution in two dimensions.Memorization <strong>of</strong> Eqs. 2/3 is not a substitute for understanding theparallelogram law and for correctly projecting a vector onto a referenceaxis. A neatly drawn sketch always helps to clarify the geometry andavoid error.F xβyF x = – F cos βF y = – F sin βyFβxF x = F sin ( π – β )F y = – F cos ( π – β )yFαβxF x = F cos( β – α )F y = F sin( β – α )Figure 2/6


c02.qxd 10/29/07 1:38 PM Page 2828 Chapter 2 Force SystemsRectangular components are convenient for finding the sum or resultantR <strong>of</strong> two <strong>force</strong>s which are concurrent. Consider two <strong>force</strong>s F 1and F 2 which are originally concurrent at a point O. Figure 2/7 showsthe line <strong>of</strong> action <strong>of</strong> F 2 shifted from O to the tip <strong>of</strong> F 1 according to thetriangle rule <strong>of</strong> Fig. 2/3. In adding the <strong>force</strong> vectors F 1 and F 2 , we maywriteorfrom which we conclude thatR F 1 F 2 (F 1xi F 1yj) (F 2xi F 2yj)R x i R y j (F 1x F 2x)i (F 1y F 2y) jR x F 1x F 2x ΣF xR y F 1y F 2y ΣF y(2/4)<strong>The</strong> term ΣF x means “the algebraic sum <strong>of</strong> the x scalar components”.For the example shown in Fig. 2/7, note that the scalar component F 2ywould <strong>be</strong> negative.© Vince Streano/Photographer’s Choice/Getty ImagesF 1yF 2yyjR yFigure 2/7OF 1xF 1F 2xF 2Rix<strong>The</strong> structural elements in the foregroundtransmit concentrated <strong>force</strong>sto the brackets at both ends.R x


c02.qxd 10/29/07 1:38 PM Page 29Article 2/3 Rectangular Components 29Sample Problem 2/1<strong>The</strong> <strong>force</strong>s F 1 , F 2 , and F 3 , all <strong>of</strong> which act on point A <strong>of</strong> the bracket, arespecified in three different ways. Determine the x and y scalar components <strong>of</strong>each <strong>of</strong> the three <strong>force</strong>s.Solution.<strong>The</strong> scalar components <strong>of</strong> F 1 , from Fig. a, areF 1x 600 cos 35 491 NF 1y 600 sin 35 344 N<strong>The</strong> scalar components <strong>of</strong> F 2 , from Fig. b, areF 2x 500 45 400 NF 2y 500 35 300 NAns.Ans.Ans.Ans.Note that the angle which orients F 2 to the x-axis is never calculated. <strong>The</strong> cosineand sine <strong>of</strong> the angle are available <strong>by</strong> inspection <strong>of</strong> the 3-4-5 triangle. Also notethat the x scalar component <strong>of</strong> F 2 is negative <strong>by</strong> inspection.<strong>The</strong> scalar components <strong>of</strong> F 3 can <strong>be</strong> obtained <strong>by</strong> first computing the angle <strong>of</strong> Fig. c.yF 2 = 500 N F 1 = 600 N3 A35°40.1 mx0.2 m0.3 mF 3 = 800 N B0.4 mFF 1 = 600 N1yFA 3x35°AF 1x(a)αF 3 = 800 N0.4 m tan 1 0.20.4 26.6<strong>The</strong>n F 3x F 3 sin 800 sin 26.6 358 NF 3y F 3 cos 800 cos 26.6 716 NAns.Ans.F 2 = 500 N34F 2x(b)AF 2yF 3y0.2 m(c)BAlternatively, the scalar components <strong>of</strong> F 3 can <strong>be</strong> obtained <strong>by</strong> writing F 3 asa magnitude times a unit vector n AB in the direction <strong>of</strong> the line segment AB.Thus,F 3 F 3 n AB F 3 ABlAB 800 0.2i 0.4j (0.2) 2 (0.4) 2<strong>The</strong> required scalar components are thenwhich agree with our previous results.F 3x 358 N 800 [0.447i 0.894j] 358i 716j NF 3y 716 NAns.Ans.Helpful Hints You should carefully examine thegeometry <strong>of</strong> each component determinationproblem and not rely onthe blind use <strong>of</strong> such formulas asF x F cos and F y F sin . A unit vector can <strong>be</strong> formed <strong>by</strong> dividingany vector, such as the geometricposition vector AB l, <strong>by</strong> itslength or magnitude. Here we usethe overarrow to denote the vectorwhich runs from A to B and theoverbar to determine the distance<strong>be</strong>tween A and B.


c02.qxd 10/29/07 1:38 PM Page 3030 Chapter 2 Force SystemsSample Problem 2/2Combine the two <strong>force</strong>s P and T, which act on the fixed structure at B, intoa single equivalent <strong>force</strong> R.P = 800 NT = 600 NBy6 mxGraphical solution. <strong>The</strong> parallelogram for the vector addition <strong>of</strong> <strong>force</strong>s T and P is constructed as shown in Fig. a. <strong>The</strong> scale used here is 1 cm 400 N; a scale<strong>of</strong> 1 cm 100 N would <strong>be</strong> more suitable for regular-size paper and would givegreater accuracy. Note that the angle a <strong>must</strong> <strong>be</strong> determined prior to construction<strong>of</strong> the parallelogram. From the given figureAα C 60°3 mDtan BDAD 6 sin 60 0.866 40.93 6 cos 60Measurement <strong>of</strong> the length R and direction <strong>of</strong> the resultant <strong>force</strong> R yields theapproximate resultsR 525 N 49Ans.T600 NαB800 NθR(a)PGeometric solution. <strong>The</strong> triangle for the vector addition <strong>of</strong> T and P is shownin Fig. b. <strong>The</strong> angle is calculated as above. <strong>The</strong> law <strong>of</strong> cosines givesR 524 NFrom the law <strong>of</strong> sines, we may determine the angle which orients R. Thus,Algebraic solution.we may writeR 2 (600) 2 (800) 2 2(600)(800) cos 40.9 274,300600sin 524sin 40.9sin 0.750 48.6Ans.Ans.By using the x-y coordinate system on the given figure,R x ΣF x 800 600 cos 40.9 346 NR y ΣF y 600 sin 40.9 393 N<strong>The</strong> magnitude and dipection <strong>of</strong> the resultant <strong>force</strong> R as shown in Fig. c are thenHelpful Hints Note the repositioning <strong>of</strong> P to permitparallelogram addition at B.BRθ800 NαT(b)600 N Note the repositioning <strong>of</strong> F so as topreserve the correct line <strong>of</strong> action <strong>of</strong>the resultant R.yαPR R2x R2y (346) 2 (393) 2 524 N tan R 1 y 393 tan1R x 346 48.6Ans.Ans.R y = – 393 N BR x = 346 Nθx<strong>The</strong> resultant R may also <strong>be</strong> written in vector notation asR R x i R y j 346i 393j NAns.(c)R


c02.qxd 10/29/07 1:38 PM Page 31Article 2/3 Rectangular Components 31Sample Problem 2/3y<strong>The</strong> 500-N <strong>force</strong> F is applied to the vertical pole as shown. (1) Write F interms <strong>of</strong> the unit vectors i and j and identify both its vector and scalar components.(2) Determine the scalar components <strong>of</strong> the <strong>force</strong> vector F along thex- and y-axes. (3) Determine the scalar components <strong>of</strong> F along the x- and y-axes.jAj′ y′30°xiSolution. Part (1). From Fig. a we may write F asF (F cos )i (F sin )j (500 cos 60)i (500 sin 60)j30°i′F = 500 Nx′ (250i 433j) NAns.yy′<strong>The</strong> scalar components are F x 250 N and F y 433 N. <strong>The</strong> vector componentsare F x 250i N and F y 433j N.Part (2). From Fig. b we may write F as F 500i N, so that the requiredscalar components areAF yθ = 60°FxAj′FF x 500 N F y 0Ans.i′Part (3). <strong>The</strong> components <strong>of</strong> F in the x- and y-directions are nonrectangularand are obtained <strong>by</strong> completing the parallelogram as shown in Fig. c. <strong>The</strong>magnitudes <strong>of</strong> the components may <strong>be</strong> calculated <strong>by</strong> the law <strong>of</strong> sines. Thus,F x sin 90 500sin 30F y sin 60 500sin 30<strong>The</strong> required scalar components are thenF x 1000 NF x 1000 NF y 866 NF y 866 NAns.(a)F xy′ F x60° 30°F90° y′90°30° 60°F = 500 NHelpful Hint(c)(b) Obtain F x and F y graphically andcompare your results with the calculatedvalues.x′xSample Problem 2/4F 1 = 100 NaForces F 1 and F 2 act on the bracket as shown. Determine the projection F b<strong>of</strong> their resultant R onto the b-axis.C30°20°F 2 = 80 NbSolution. <strong>The</strong> parallelogram addition <strong>of</strong> F 1 and F 2 is shown in the figure.Using the law <strong>of</strong> cosines gives usR 2 (80) 2 (100) 2 2(80)(100) cos 130<strong>The</strong> figure also shows the orthogonal projection F b <strong>of</strong> R onto the b-axis. Itslength isF b 80 100 cos 50 144.3 NR 163.4 NAns.C100 NF 150°F 280 NR50°aNote that the components <strong>of</strong> a vector are in general not equal to the projections<strong>of</strong> the vector onto the same axes. If the a-axis had <strong>be</strong>en perpendicular tothe b-axis, then the projections and components <strong>of</strong> R would have <strong>be</strong>en equal.F bb


c02.qxd 10/29/07 1:38 PM Page 3232 Chapter 2 Force SystemsPROBLEMSyIntroductory ProblemsF = 4.8 kN2/1 <strong>The</strong> <strong>force</strong> F has a magnitude <strong>of</strong> 800 N. Express F as avector in terms <strong>of</strong> the unit vectors i and j. Identify thex and y scalar components <strong>of</strong> F.Ans. F 459i 655j NF x 459 N, F y 655 NyO34xF = 800 N35°Problem 2/3Ox2/4 <strong>The</strong> line <strong>of</strong> action <strong>of</strong> the 9.6-kN <strong>force</strong> F runs throughthe points A and B as shown in the figure. Determinethe x and y scalar components <strong>of</strong> F.y, mmProblem 2/1B (300, 100)2/2 <strong>The</strong> magnitude <strong>of</strong> the <strong>force</strong> F is 600 N. Express F as avector in terms <strong>of</strong> the unit vectors i and j. Identifyboth the scalar and vector components <strong>of</strong> F.OF = 9.6 kNx, mmy, mA (–150, –200)A (–3, 1)Problem 2/430°OF = 600 Nx, m2/5 A cable stretched <strong>be</strong>tween the fixed supports A and Bis under a tension T <strong>of</strong> 900 N. Express the tension asa vector using the unit vectors i and j, first, as a <strong>force</strong>T A acting on A and second, as a <strong>force</strong> T B acting on B.Ans. T A 749i 499j N, T B 749i 499j NProblem 2/2A3 m2/3 <strong>The</strong> slope <strong>of</strong> the 4.8-kN <strong>force</strong> F is specified as shownin the figure. Express F as a vector in terms <strong>of</strong> theunit vectors i and j.Ans. F 2.88i 3.84j kNy2 mxBProblem 2/5


c02.qxd 10/29/07 1:38 PM Page 33Article 2/3 Problems 332/6 <strong>The</strong> 1800-N <strong>force</strong> F is applied to the end <strong>of</strong> the I-<strong>be</strong>am. Express F as a vector using the unit vectors iand j.F 1 = 800 NyF 2 = 425 NyF = 1800 N70°θA3O4xzxzProblem 2/62/7 <strong>The</strong> two structural mem<strong>be</strong>rs, one <strong>of</strong> which is in tensionand the other in compression, exert the indicated<strong>force</strong>s on joint O. Determine the magnitude <strong>of</strong> the resultantR <strong>of</strong> the two <strong>force</strong>s and the angle which Rmakes with the positive x-axis.Ans. R 3.61 kN, 206Representative ProblemsProblem 2/82 kN3 kN2/9 In the design <strong>of</strong> a control mechanism, it is determinedthat rod AB transmits a 260-N <strong>force</strong> P to the crankBC. Determine the x and y scalar components <strong>of</strong> P.Ans. P x 240 NP y 100 NP = 260 N30° 60°OxtBA125Problem 2/72/8 Two <strong>force</strong>s are applied to the construction bracket asshown. Determine the angle which makes the resultant<strong>of</strong> the two <strong>force</strong>s vertical. Determine the magnitudeR <strong>of</strong> the resultant.nCy30°xProblem 2/92/10 For the mechanism <strong>of</strong> Prob. 2/9, determine thescalar components P t and P n <strong>of</strong> P which are tangentand normal, respectively, to crank BC.


c02.qxd 10/29/07 1:38 PM Page 3434 Chapter 2 Force Systems2/11 <strong>The</strong> t-component <strong>of</strong> the <strong>force</strong> F is known to <strong>be</strong> 75N. Determine the n-component and the magnitude<strong>of</strong> F.Ans. F n 62.9 N, F 97.9 NntF 1 = 3 kN30°AF10°45°F 2 = 7 kN30°yProblem 2/112/12 A <strong>force</strong> F <strong>of</strong> magnitude 800 N is applied to point C <strong>of</strong>the bar AB as shown. Determine both the x-y andthe n-t components <strong>of</strong> F.AynxProblem 2/132/14 To satisfy design limitations it is necessary to determinethe effect <strong>of</strong> the 2-kN tension in the cable onthe shear, tension, and <strong>be</strong>nding <strong>of</strong> the fixed I-<strong>be</strong>am.For this purpose replace this <strong>force</strong> <strong>by</strong> its equivalent<strong>of</strong> two <strong>force</strong>s at A, F t parallel and F n perpendicularto the <strong>be</strong>am. Determine F t and F n .xtAC20°40°F = 800 N30°2 kN60°BProblem 2/122/13 <strong>The</strong> two <strong>force</strong>s shown act at point A <strong>of</strong> the <strong>be</strong>nt bar.Determine the resultant R <strong>of</strong> the two <strong>force</strong>s.Ans. R 2.35i 3.45j kNProblem 2/14


c02.qxd 10/29/07 1:38 PM Page 35Article 2/3 Problems 352/15 Determine the magnitude F s <strong>of</strong> the tensile spring<strong>force</strong> in order that the resultant <strong>of</strong> F s and F is a vertical<strong>force</strong>. Determine the magnitude R <strong>of</strong> this verticalresultant <strong>force</strong>.Ans. F s 250 N, R 433 NbF = 2 kNF = 500 N60°45°Problem 2/17aA60°2/18 Determine the scalar components R a and R b <strong>of</strong> the<strong>force</strong> R along the nonrectangular axes a and b. Alsodetermine the orthogonal projection P a <strong>of</strong> R ontoaxis a.Problem 2/15a2/16 <strong>The</strong> ratio <strong>of</strong> the lift <strong>force</strong> L to the drag <strong>force</strong> D forthe simple airfoil is L/D 10. If the lift <strong>force</strong> on ashort section <strong>of</strong> the airfoil is 200 N, compute themagnitude <strong>of</strong> the resultant <strong>force</strong> R and the angle which it makes with the horizontal.O30°R = 800 NL110°bAir flowCDProblem 2/16Problem 2/182/19 Determine the resultant R <strong>of</strong> the two <strong>force</strong>s shown<strong>by</strong> (a) applying the parallelogram rule for vector additionand (b) summing scalar components.Ans. R 520i 700j N2/17 Determine the components <strong>of</strong> the 2-kN <strong>force</strong> alongthe oblique axes a and b. Determine the projections<strong>of</strong> F onto the a- and b-axes.Ans. F a 0.598 kN, F b 1.633 kNP a 1.414 kN, P b 1.932 kNyx60°600 N400 NProblem 2/19


c02.qxd 10/29/07 1:38 PM Page 3636 Chapter 2 Force Systems2/20 It is desired to remove the spike from the tim<strong>be</strong>r <strong>by</strong>applying <strong>force</strong> along its horizontal axis. An obstructionA prevents direct access, so that two <strong>force</strong>s, one1.6 kN and the other P, are applied <strong>by</strong> cables asshown. Compute the magnitude <strong>of</strong> P necessary toensure a resultant T directed along the spike. Als<strong>of</strong>ind T.yxO60 mmA40 mm200 mmP100 mm80 mmθnktA150 mmPProblem 2/22Problem 2/202/21 At what angle <strong>must</strong> the 800-N <strong>force</strong> <strong>be</strong> applied inorder that the resultant R <strong>of</strong> the two <strong>force</strong>s has amagnitude <strong>of</strong> 2000 N? For this condition, determinethe angle <strong>be</strong>tween R and the vertical.Ans. 51.3, 18.191400 Nθ1.6 kN800 N2/23 Refer to the statement and figure <strong>of</strong> Prob. 2/22.When pin P is in the position 20, determine then- and t-components <strong>of</strong> the <strong>force</strong> F which the spring<strong>of</strong> modulus k 1.2 kN/m exerts on the pin.Ans. F n 19.18 N, F t 13.84 N2/24 <strong>The</strong> cable AB prevents bar OA from rotating clockwiseabout the pivot O. If the cable tension is 750 N,determine the n- and t-components <strong>of</strong> this <strong>force</strong> actingon point A <strong>of</strong> the bar.nAtBO60°1.5 mProblem 2/212/22 <strong>The</strong> unstretched length <strong>of</strong> the spring <strong>of</strong> modulus k 1.2 kN/m is 100 mm. When pin P is in the position 30, determine the x- and y-components <strong>of</strong> the<strong>force</strong> which the spring exerts on the pin. (<strong>The</strong> <strong>force</strong>in a spring is given <strong>by</strong> F kx, where x is the deflectionfrom the unstretched length.)1.2 mProblem 2/24


c02.qxd 10/29/07 1:38 PM Page 37Article 2/3 Problems 372/25 At what angle <strong>must</strong> the 400-N <strong>force</strong> <strong>be</strong> applied inorder that the resultant R <strong>of</strong> the two <strong>force</strong>s have amagnitude <strong>of</strong> 1000 N? For this condition what will<strong>be</strong> the angle <strong>be</strong>tween R and the horizontal?Ans. 51.3, 18.19400 N2/27 <strong>The</strong> guy cables AB and AC are attached to the top <strong>of</strong>the transmission tower. <strong>The</strong> tension in cable AC is 8kN. Determine the required tension T in cable ABsuch that the net effect <strong>of</strong> the two cable tensions is adownward <strong>force</strong> at point A. Determine the magnitudeR <strong>of</strong> this downward <strong>force</strong>.Ans. T 5.68 kN, R 10.21 kNA40 m700 NθBO20 mC50 m40 mProblem 2/252/26 In the design <strong>of</strong> the robot to insert the small cylindricalpart into a close-fitting circular hole, the robotarm <strong>must</strong> exert a 90-N <strong>force</strong> P on the part parallel tothe axis <strong>of</strong> the hole as shown. Determine the components<strong>of</strong> the <strong>force</strong> which the part exerts on the robotalong axes (a) parallel and perpendicular to the armAB, and (b) parallel and perpendicular to the arm BC.Problem 2/272/28 <strong>The</strong> gusset plate is subjected to the two <strong>force</strong>sshown. Replace them <strong>by</strong> two equivalent <strong>force</strong>s, F x inthe x-direction and F a in the a-direction. Determinethe magnitudes <strong>of</strong> F x and F a . Solve geometrically orgraphically.B15°yA60°45°CDP = 90 NA800 N10°x900 N25° 45°aProblem 2/28Problem 2/26


c02.qxd 10/29/07 1:38 PM Page 3838 Chapter 2 Force SystemsdF2/4 MOMENTIn addition to the tendency to move a body in the direction <strong>of</strong> its application,a <strong>force</strong> can also tend to rotate a body about an axis. <strong>The</strong> axismay <strong>be</strong> any line which neither intersects nor is parallel to the line <strong>of</strong> action<strong>of</strong> the <strong>force</strong>. This rotational tendency is known as the moment M <strong>of</strong>the <strong>force</strong>. Moment is also referred to as torque.As a familiar example <strong>of</strong> the concept <strong>of</strong> moment, consider the pipewrench <strong>of</strong> Fig. 2/8a. One effect <strong>of</strong> the <strong>force</strong> applied perpendicular tothe handle <strong>of</strong> the wrench is the tendency to rotate the pipe about itsvertical axis. <strong>The</strong> magnitude <strong>of</strong> this tendency depends on both themagnitude F <strong>of</strong> the <strong>force</strong> and the effective length d <strong>of</strong> the wrenchhandle. Common experience shows that a pull which is not perpendicularto the wrench handle is less effective than the right-angle pullshown.Moment about a PointAOMdr(a)FαFigure 2/8b shows a two-dimensional body acted on <strong>by</strong> a <strong>force</strong> F inits plane. <strong>The</strong> magnitude <strong>of</strong> the moment or tendency <strong>of</strong> the <strong>force</strong> to rotatethe body about the axis O-O perpendicular to the plane <strong>of</strong> thebody is proportional both to the magnitude <strong>of</strong> the <strong>force</strong> and to the momentarm d, which is the perpendicular distance from the axis to theline <strong>of</strong> action <strong>of</strong> the <strong>force</strong>. <strong>The</strong>refore, the magnitude <strong>of</strong> the moment isdefined asM Fd(2/5)+yO (b)M(c)xM = FdAd(d)Figure 2/8F<strong>The</strong> moment is a vector M perpendicular to the plane <strong>of</strong> the body. <strong>The</strong>sense <strong>of</strong> M depends on the direction in which F tends to rotate thebody. <strong>The</strong> right-hand rule, Fig. 2/8c, is used to identify this sense. Werepresent the moment <strong>of</strong> F about O-O as a vector pointing in the direction<strong>of</strong> the thumb, with the fingers curled in the direction <strong>of</strong> the rotationaltendency.<strong>The</strong> moment M o<strong>be</strong>ys all the rules <strong>of</strong> vector combination and may<strong>be</strong> considered a sliding vector with a line <strong>of</strong> action coinciding with themoment axis. <strong>The</strong> basic units <strong>of</strong> moment in SI units are newton-meters(N m), and in the U.S. customary system are pound-feet (lb-ft).When dealing with <strong>force</strong>s which all act in a given plane, we customarilyspeak <strong>of</strong> the moment about a point. By this we mean the momentwith respect to an axis normal to the plane and passing through thepoint. Thus, the moment <strong>of</strong> <strong>force</strong> F about point A in Fig. 2/8d has themagnitude M Fd and is counterclockwise.Moment directions may <strong>be</strong> accounted for <strong>by</strong> using a stated sign convention,such as a plus sign () for counterclockwise moments and aminus sign () for clockwise moments, or vice versa. Sign consistencywithin a given problem is essential. For the sign convention <strong>of</strong> Fig. 2/8d,the moment <strong>of</strong> F about point A (or about the z-axis passing throughpoint A) is positive. <strong>The</strong> curved arrow <strong>of</strong> the figure is a convenient wayto represent moments in two-dimensional analysis.


c02.qxd 10/29/07 1:38 PM Page 39Article 2/4 Moment 39<strong>The</strong> Cross ProductIn some two-dimensional and many <strong>of</strong> the three-dimensional problemsto follow, it is convenient to use a vector approach for moment calculations.<strong>The</strong> moment <strong>of</strong> F about point A <strong>of</strong> Fig. 2/8b may <strong>be</strong>represented <strong>by</strong> the cross-product expressionM r F(2/6)where r is a position vector which runs from the moment referencepoint A to any point on the line <strong>of</strong> action <strong>of</strong> F. <strong>The</strong> magnitude <strong>of</strong> this expressionis given <strong>by</strong>*M Fr sin Fd(2/7)which agrees with the moment magnitude as given <strong>by</strong> Eq. 2/5. Note thatthe moment arm d r sin does not depend on the particular point onthe line <strong>of</strong> action <strong>of</strong> F to which the vector r is directed. We establish thedirection and sense <strong>of</strong> M <strong>by</strong> applying the right-hand rule to the sequencer F. If the filgers <strong>of</strong> the right hand are curled in the direction<strong>of</strong> rotation from the positive sense <strong>of</strong> r to the positive sense <strong>of</strong> F, thenthe thumb points in the positive sense <strong>of</strong> M.We <strong>must</strong> maintain the sequence r F, <strong>be</strong>cause the sequence F rwould produce a vector with a sense opposite to that <strong>of</strong> the correctmoment. As was the case with the scalar approach, the moment Mmay <strong>be</strong> thought <strong>of</strong> as the moment about point A or as the momentabout the line O-O which passes through point A and is perpendicularto the plane containing the vectors r and F. When we evaluate themoment <strong>of</strong> a <strong>force</strong> about a given point, the choice <strong>be</strong>tween using thevector cross product or the scalar expression depends on how thegeometry <strong>of</strong> the problem is specified. If we know or can easily determinethe perpendicular distance <strong>be</strong>tween the line <strong>of</strong> action <strong>of</strong> the<strong>force</strong> and the moment center, then the scalar approach is generallysimpler. If, however, F and r are not perpendicular and are easily expressiblein vector notation, then the cross-product expression is<strong>of</strong>ten preferable.In Section B <strong>of</strong> this chapter, we will see how the vector formulation<strong>of</strong> the moment <strong>of</strong> a <strong>force</strong> is especially useful for determining the moment<strong>of</strong> a <strong>force</strong> about a point in three-dimensional situations.Varignon’s <strong>The</strong>oremOne <strong>of</strong> the most useful principles <strong>of</strong> mechanics is Varignon’s theorem,which states that the moment <strong>of</strong> a <strong>force</strong> about any point is equal tothe sum <strong>of</strong> the moments <strong>of</strong> the components <strong>of</strong> the <strong>force</strong> about the samepoint.*See item 7 in Art. C/7 <strong>of</strong> Appendix C for additional information concerning the crossproduct.


c02.qxd 10/29/07 1:38 PM Page 4040 Chapter 2 Force SystemsTo prove this theorem, consider the <strong>force</strong> R acting in the plane <strong>of</strong>the body shown in Fig. 2/9a. <strong>The</strong> <strong>force</strong>s P and Q represent any two nonrectangularcomponents <strong>of</strong> R. <strong>The</strong> moment <strong>of</strong> R about point O isBecause R P Q, we may writeM O r Rr R r (P Q)Using the distributive law for cross products, we haveM O r R r P r Q(2/8)which says that the moment <strong>of</strong> R about O equals the sum <strong>of</strong> the momentsabout O <strong>of</strong> its components P and Q. This proves the theorem.Varignon’s theorem need not <strong>be</strong> restricted to the case <strong>of</strong> two components,but it applies equally well to three or more. Thus we could haveused any num<strong>be</strong>r <strong>of</strong> concurrent components <strong>of</strong> R in the foregoingpro<strong>of</strong>.*Figure 2/9b illustrates the usefulness <strong>of</strong> Varignon’s theorem. <strong>The</strong>moment <strong>of</strong> R about point O is Rd. However, if d is more difficult to determinethan p and q, we can resolve R into the components P and Q,and compute the moment asM O Rd pP qQwhere we take the clockwise moment sense to <strong>be</strong> positive.Sample Problem 2/5 shows how Varignon’s theorem can help us tocalculate moments.PRPRBQqBQOrdOp(a)(b)Figure 2/9*As originally stated, Varignon’s theorem was limited to the case <strong>of</strong> two concurrent components<strong>of</strong> a given <strong>force</strong>. See <strong>The</strong> Science <strong>of</strong> Mechanics, <strong>by</strong> Ernst Mach, originally publishedin 1883.


c02.qxd 10/29/07 1:38 PM Page 4242 Chapter 2 Force SystemsSample Problem 2/6TB<strong>The</strong> trap door OA is raised <strong>by</strong> the cable AB, which passes over the small frictionlessguide pulleys at B. <strong>The</strong> tension everywhere in the cable is T, and this tensionapplied at A causes a moment M O about the hinge at O. Plot the quantity M O /Tas a function <strong>of</strong> the door elevation angle over the range 0 90 and note minimumand maximum values. What is the physical significance <strong>of</strong> this ratio?0.4 mO0.5 mθ0.3 mASolution. We <strong>be</strong>gin <strong>by</strong> constructing a figure which shows the tension <strong>force</strong> Tacting directly on the door, which is shown in an arbitrary angular position . Itshould <strong>be</strong> clear that the direction <strong>of</strong> T will vary as varies. In order to deal withthis variation, we write a unit vector n AB which “aims” T:Using the x-y coordinates <strong>of</strong> our figure, we can writeSoand<strong>The</strong> desired unit vector isOur tension vector can now <strong>be</strong> written as<strong>The</strong> moment <strong>of</strong> T about point O, as a vector, is M O r OB T, where r OB 0.4j m, or<strong>The</strong> magnitude <strong>of</strong> M O isand the requested ratio isn AB r ABr AB r OB r OAr ABr OB 0.4j m and r OA 0.5(cos i sin j) mr AB r OB r OA 0.4j (0.5)(cos i sin j) 0.5 cos i (0.4 0.5 sin )j mr AB (0.5 cos ) 2 (0.4 0.5 sin ) 2 0.41 0.4 sin mn AB r AB 0.5 cos i (0.4 0.5 sin )jrAB 0.41 0.4 sin T Tn AB T 0.5 cos i (0.4 0.5 sin )j0.41 0.4 sin M O 0.4j T 0.5 cos i (0.4 0.5 sin )j0.41 0.4 sin 0.2T cos 0.41 0.4 sin k0.2T cos M O 0.41 0.4 sin M OT 0.2 cos 0.41 0.4 sin Ans.which is plotted in the accompanying graph. <strong>The</strong> expression M O /T is the momentarm d (in meters) which runs from O to the line <strong>of</strong> action <strong>of</strong> T. It has a maximumvalue <strong>of</strong> 0.4 m at 53.1 (at which point T is horizontal) and a minimum value <strong>of</strong>0 at 90 (at which point T is vertical). <strong>The</strong> expression is valid even if T varies.This sample problem treats moments in two-dimensional <strong>force</strong> <strong>systems</strong>, andit also points out the advantages <strong>of</strong> carrying out a solution for an arbitrary position,so that <strong>be</strong>havior over a range <strong>of</strong> positions can <strong>be</strong> examined.Br OBOyd0.50.40.3M O——–, mT 0.20.1r ABr OATθAHelpful Hints Recall that any unit vector can <strong>be</strong>written as a vector divided <strong>by</strong> itsmagnitude. In this case the vector inthe numerator is a position vector.x00 10 20 30 40 50 60 70 80 90θ, deg Recall that any vector may <strong>be</strong> writtenas a magnitude times an “aiming”unit vector. In the expression M r F, the positionvector r runs from the momentcenter to any point on the line<strong>of</strong> action <strong>of</strong> F. Here, r OB is more convenientthan r OA .


c02.qxd 10/29/07 1:38 PM Page 43Article 2/4 Problems 43PROBLEMSIntroductory Problems2/29 <strong>The</strong> 10-kN <strong>force</strong> is applied at point A. Determine themoment <strong>of</strong> F about point O. Determine the points onthe x- and y-axes about which the moment <strong>of</strong> F iszero.Ans. M O 16 kN m CW(x, y) (2.67, 0) m and (0, 2) m2/31 Determine the moment <strong>of</strong> the 50-N <strong>force</strong> (a) aboutpoint O <strong>by</strong> Varignon’s theorem and (b) about point C<strong>by</strong> a vector approach.Ans. M O 519 N mm CCW, M C 1616k N mmy, mmC (0, 25)B (40, 10)A (–4, 5)y, mOF = 50 Nx, mm34F = 10 kNA (–15, –20)OProblem 2/29x, mProblem 2/312/32 <strong>The</strong> <strong>force</strong> <strong>of</strong> magnitude F acts along the edge <strong>of</strong> thetriangular plate. Determine the moment <strong>of</strong> F aboutpoint O.2/30 Determine the moment <strong>of</strong> the 800-N <strong>force</strong> aboutpoint A and about point O.hFAy875 mmBObProblem 2/32625 mmOF = 800 N 30°x2/33 In steadily turning the water pump, a person exertsthe 120-N <strong>force</strong> on the handle as shown. Determinethe moment <strong>of</strong> this <strong>force</strong> about point O.Ans. M O 14.74 N m CW15°F = 120 NAProblem 2/3020°150 mmOProblem 2/33


c02.qxd 10/29/07 1:38 PM Page 4444 Chapter 2 Force Systems2/34 <strong>The</strong> throttle-control sector pivots freely at O. If aninternal torsional spring exerts a return moment M 1.8 N m on the sector when in the positionshown, for design purposes determine the necessarythrottle-cable tension T so that the net momentabout O is zero. Note that when T is zero, the sectorrests against the idle-control adjustment screw at R.30 mm200 mm15°250 NQTPProblem 2/36R50 mm2/37 A mechanic pulls on the 13-mm combination wrenchwith the 140-N <strong>force</strong> shown. Determine the moment<strong>of</strong> this <strong>force</strong> about the bolt center O.Ans. M O 13.10 N m CCWMOF = 140 N25°Problem 2/34O15°2/35 A <strong>force</strong> F <strong>of</strong> magnitude 60 N is applied to the gear.Determine the moment <strong>of</strong> F about point O.Ans. M O 5.64 N m CW95 mmAOr = 100 mm20°F = 60 NProblem 2/372/38 As a trailer is towed in the forward direction, the<strong>force</strong> F 500 N is applied as shown to the ball <strong>of</strong> thetrailer hitch. Determine the moment <strong>of</strong> this <strong>force</strong>about point O.F = 500 NProblem 2/3530°2/36 Calculate the moment <strong>of</strong> the 250-N <strong>force</strong> on the handle<strong>of</strong> the monkey wrench about the center <strong>of</strong> thebolt.32 mmAO38 mm275 mmProblem 2/38


c02.qxd 10/29/07 1:38 PM Page 45Article 2/4 Problems 45Representative Problems2/39 A portion <strong>of</strong> a mechanical coin sorter works as follows:Pennies and dimes roll down the 20 incline,the last triangular portion <strong>of</strong> which pivots freelyabout a horizontal axis through O. Dimes are lightenough (2.28 grams each) so that the triangular portionremains stationary, and the dimes roll into theright collection column. Pennies, on the other hand,are heavy enough (3.06 grams each) so that the triangularportion pivots clockwise, and the penniesroll into the left collection column. Determine themoment about O <strong>of</strong> the weight <strong>of</strong> the penny in terms<strong>of</strong> the slant distance s in millimeters.Ans. M O 0.1335 0.0282s N mm (s in mm)2/40 Elements <strong>of</strong> the lower arm are shown in the figure.<strong>The</strong> mass <strong>of</strong> the forearm is 2.3 kg with mass centerat G. Determine the combined moment about theelbow pivot O <strong>of</strong> the weights <strong>of</strong> the forearm and the3.6-kg homogeneous sphere. What <strong>must</strong> the bicepstension <strong>force</strong> <strong>be</strong> so that the overall moment about Ois zero?TsO50 mm3.5 mmLIBERTYTRUSTWE GOD IN INO 20°19899.5 mm55°150 mmGA2.3(9.81) N325 mm3.6(9.81) NProblem 2/40penniesProblem 2/39dimes2/41 A 150-N pull T is applied to a cord, which is woundsecurely around the inner hub <strong>of</strong> the drum. Determinethe moment <strong>of</strong> T about the drum center C. Atwhat angle should T <strong>be</strong> applied so that the momentabout the contact point P is zero?Ans. M C 18.75 N m CW, 51.3T = 150 Nθ125 mmC200 mmPProblem 2/41


c02.qxd 10/29/07 1:38 PM Page 4646 Chapter 2 Force Systems2/42 A <strong>force</strong> <strong>of</strong> 200 N is applied to the end <strong>of</strong> the wrenchto tighten a flange bolt which holds the wheel to theaxle. Determine the moment M produced <strong>by</strong> this<strong>force</strong> about the center O <strong>of</strong> the wheel for the position<strong>of</strong> the wrench shown.450 mm200 N2/44 <strong>The</strong> uniform work platform, which has a mass perunit length <strong>of</strong> 28 kg/m, is simply supported <strong>by</strong> crossrods A and B. <strong>The</strong> 90-kg construction worker startsfrom point B and walks to the right. At what locations will the combined moment <strong>of</strong> the weights <strong>of</strong>the man and platform about point B <strong>be</strong> zero?1 m 4 m 3 ms90 kg650 mmO20°28 kg/m125 mmABProblem 2/422/43 In order to raise the flagpole OC, a light frame OABis attached to the pole and a tension <strong>of</strong> 3.2 kN is developedin the hoisting cable <strong>by</strong> the power winch D.Calculate the moment M O <strong>of</strong> this tension about thehinge point O.Ans. M O 6.17 kN m CCWAProblem 2/442/45 In raising the pole from the position shown, the tensionT in the cable <strong>must</strong> supply a moment about O <strong>of</strong>72 kN m. Determine T.Ans. T 8.65 kN10 mD20°3 m 3 m3 m BOCT30 mProblem 2/4312 mO60°Problem 2/45


c02.qxd 10/29/07 1:38 PM Page 47Article 2/4 Problems 472/46 <strong>The</strong> <strong>force</strong> exerted <strong>by</strong> the plunger <strong>of</strong> cylinder AB onthe door is 40 N directed along the line AB, and this<strong>force</strong> tends to keep the door closed. Compute the moment<strong>of</strong> this <strong>force</strong> about the hinge O. What <strong>force</strong> F Cnormal to the plane <strong>of</strong> the door <strong>must</strong> the door stopat C exert on the door so that the combined momentabout O <strong>of</strong> the two <strong>force</strong>s is zero?2/48 Calculate the moment M A <strong>of</strong> the 200-N <strong>force</strong> aboutpoint A <strong>by</strong> using three scalar methods and one vectormethod.200 N15°BOA B75100 C400 40025280mmAyx400 mmProblem 2/48Dimensions in millimetersProblem 2/462/47 <strong>The</strong> 10-N <strong>force</strong> is applied to the handle <strong>of</strong> the hydrauliccontrol valve as shown. Calculate the moment<strong>of</strong> this <strong>force</strong> about point O.Ans. M O 2.81 N m CWA2/49 An exerciser <strong>be</strong>gins with his arm in the relaxed verticalposition OA, at which the elastic band is unstretched.He then rotates his arm to the horizontalposition OB. <strong>The</strong> elastic modulus <strong>of</strong> the band is k 60 N/m—that is, 60 N <strong>of</strong> <strong>force</strong> is required to stretchthe band each additional meter <strong>of</strong> elongation. Determinethe moment about O <strong>of</strong> the <strong>force</strong> which theband exerts on the hand B.Ans. M O 26.8 N m CCW20°F = 10 N250 mmB635 mmO60°37.5 mmBOFProblem 2/47A740 mmCProblem 2/49


c02.qxd 10/29/07 1:38 PM Page 4848 Chapter 2 Force Systems2/50 (a) Calculate the moment <strong>of</strong> the 90-N <strong>force</strong> aboutpoint O for the condition 15. Also, determinethe value <strong>of</strong> for which the moment about O is (b)zero and (c) a maximum.2/52 Design criteria require that the robot exert the 90-N<strong>force</strong> on the part as shown while inserting a cylindricalpart into the circular hole. Determine the momentabout points A, B, and C <strong>of</strong> the <strong>force</strong> which thepart exerts on the robot.O150 mmB15°800 mmF = 90 N600 mmθAA550 mm450 mm60°45°CDP = 90 NProblem 2/502/51 <strong>The</strong> small crane is mounted along the side <strong>of</strong> apickup <strong>be</strong>d and facilitates the handling <strong>of</strong> heavyloads. When the boom elevation angle is 40, the<strong>force</strong> in the hydraulic cylinder BC is 4.5 kN, and this<strong>force</strong> applied at point C is in the direction from B toC (the cylinder is in compression). Determine themoment <strong>of</strong> this 4.5-kN <strong>force</strong> about the boom pivotpoint O.Ans. M O 0.902 kN m CWProblem 2/522/53 <strong>The</strong> masthead fitting supports the two <strong>force</strong>s shown.Determine the magnitude <strong>of</strong> T which will cause no<strong>be</strong>nding <strong>of</strong> the mast (zero moment) at point O.Ans. T 4.04 kNA90mm120mm785mm110mmCθ340mmO360mmC60 mm 2O30°5B5 kNTProblem 2/51Problem 2/53


c02.qxd 10/29/07 1:38 PM Page 49Article 2/4 Problems 492/54 <strong>The</strong> piston, connecting rod, and crankshaft <strong>of</strong> adiesel engine are shown in the figure. <strong>The</strong> crankthrow OA is half the stroke <strong>of</strong> 200 mm, and thelength AB <strong>of</strong> the rod is 350 mm. For the position indicated,the rod is under a compression along AB <strong>of</strong>16 kN. Determine the moment M <strong>of</strong> this <strong>force</strong> aboutthe crankshaft axis O.B2/56 If the combined moment <strong>of</strong> the two <strong>force</strong>s aboutpoint C is zero, determine(a) the magnitude <strong>of</strong> the <strong>force</strong> P(b) the magnitude R <strong>of</strong> the resultant <strong>of</strong> the tw<strong>of</strong>orces(c) the coordinates x and y <strong>of</strong> the point A on the rim<strong>of</strong> the wheel about which the combined moment<strong>of</strong> the two <strong>force</strong>s is a maximum(d) the combined moment M A <strong>of</strong> the two <strong>force</strong>sabout A.Ans. (a) P 61.6 N(b) R 141.3 N(c) x 49.0 mm, y 63.2 mm(d) M A 15.77 N m CWyOA30°100 N60°OC40 mmP40 mmx40 mmProblem 2/54Problem 2/562/55 <strong>The</strong> 120-N <strong>force</strong> is applied as shown to one end <strong>of</strong>the curved wrench. If 30, calculate the moment<strong>of</strong> F about the center O <strong>of</strong> the bolt. Determine thevalue <strong>of</strong> which would maximize the moment aboutO; state the value <strong>of</strong> this maximum moment.Ans. M O 41.5 N m CW 33.2, (M O ) max 41.6 N m CWF = 120 Nα25 mmA150 mm70mm70mm25 mmOProblem 2/55


c02.qxd 10/29/07 1:38 PM Page 5050 Chapter 2 Force SystemsOOOad(a)r Ar B(b)(c)–FFB –FrFAM2/5 COUPLE<strong>The</strong> moment produced <strong>by</strong> two equal, opposite, and noncollinear<strong>force</strong>s is called a couple. Couples have certain unique <strong>properties</strong> andhave important applications in mechanics.Consider the action <strong>of</strong> two equal and opposite <strong>force</strong>s F and F a distanced apart, as shown in Fig. 2/10a. <strong>The</strong>se two <strong>force</strong>s cannot <strong>be</strong> combinedinto a single <strong>force</strong> <strong>be</strong>cause their sum in every direction is zero. <strong>The</strong>ir onlyeffect is to produce a tendency <strong>of</strong> rotation. <strong>The</strong> combined moment <strong>of</strong> thetwo <strong>force</strong>s about an axis normal to their plane and passing through anypoint such as O in their plane is the couple M. This couple has a magnitudeorM F(a d) FaM FdIts direction is counterclockwise when viewed from above for the case illustrated.Note especially that the magnitude <strong>of</strong> the couple is independent<strong>of</strong> the distance a which locates the <strong>force</strong>s with respect to themoment center O. It follows that the moment <strong>of</strong> a couple has the samevalue for all moment centers.Vector Algebra MethodWe may also express the moment <strong>of</strong> a couple <strong>by</strong> using vector algebra.With the cross-product notation <strong>of</strong> Eq. 2/6, the combined momentabout point O <strong>of</strong> the <strong>force</strong>s forming the couple <strong>of</strong> Fig. 2/10b isMMM r A F r B (F) (r A r B ) Fwhere r A and r B are position vectors which run from point O to arbitrarypoints A and B on the lines <strong>of</strong> action <strong>of</strong> F and F, respectively. Becauser A r B r, we can express M asM r FMCounterclockwisecouple(d)Figure 2/10MClockwisecoupleHere again, the moment expression contains no reference to the momentcenter O and, therefore, is the same for all moment centers. Thus,we may represent M <strong>by</strong> a free vector, as show in Fig. 2/10c, where thedirection <strong>of</strong> M is normal to the plane <strong>of</strong> the couple and the sense <strong>of</strong> M isestablished <strong>by</strong> the right-hand rule.Because the couple vector M is always perpendicular to the plane <strong>of</strong>the <strong>force</strong>s which constitute the couple, in two-dimensional analysis wecan represent the sense <strong>of</strong> a couple vector as clockwise or counterclockwise<strong>by</strong> one <strong>of</strong> the conventions shown in Fig. 2/10d. Later, when we dealwith couple vectors in three-dimensional problems, we will make fulluse <strong>of</strong> vector notation to represent them, and the mathematics will automaticallyaccount for their sense.Equivalent CouplesChanging the values <strong>of</strong> F and d does not change a given couple aslong as the product Fd remains the same. Likewise, a couple is not affectedif the <strong>force</strong>s act in a different but parallel plane. Figure 2/11


c02.qxd 10/29/07 1:38 PM Page 5252 Chapter 2 Force SystemsSample Problem 2/7<strong>The</strong> rigid structural mem<strong>be</strong>r is subjected to a couple consisting <strong>of</strong> the two100-N <strong>force</strong>s. Replace this couple <strong>by</strong> an equivalent couple consisting <strong>of</strong> the tw<strong>of</strong>orces P and P, each <strong>of</strong> which has a magnitude <strong>of</strong> 400 N. Determine the properangle .40M–P θ θPSolution. <strong>The</strong> original couple is counterclockwise when the plane <strong>of</strong> the <strong>force</strong>sis viewed from above, and its magnitude is[M Fd]M 100(0.1) 10 N m100100<strong>The</strong> <strong>force</strong>s P and P produce a counterclockwise coupleM 400(0.040) cos 100 N10060Equating the two expressions gives100 NHelpful Hint10 (400)(0.040) cos cos1 1016 51.3Ans. Since the two equal couples are parallel free vectors, the only dimensionswhich are relevant are those which give the perpendicular distances <strong>be</strong>tweenthe <strong>force</strong>s <strong>of</strong> the couples.Dimensions in millimetersP = 400 Nθ40 mmdθθP = 400 NSample Problem 2/8Replace the horizontal 400-N <strong>force</strong> acting on the lever <strong>by</strong> an equivalent systemconsisting <strong>of</strong> a <strong>force</strong> at O and a couple.200 mm400 NSolution. We apply two equal and opposite 400-N <strong>force</strong>s at O and identify thecounterclockwise coupleO60°[M Fd]M 400(0.200 sin 60) 69.3 N mThus, the original <strong>force</strong> is equivalent to the 400-N <strong>force</strong> at O and thecouple as shown in the third <strong>of</strong> the three equivalent figures.Ans.69.3-N mHelpful Hint <strong>The</strong> reverse <strong>of</strong> this problem is <strong>of</strong>ten encountered, namely, the replacement<strong>of</strong> a <strong>force</strong> and a couple <strong>by</strong> a single <strong>force</strong>. Proceeding in reverse is the same asreplacing the couple <strong>by</strong> two <strong>force</strong>s, one <strong>of</strong> which is equal and opposite to the400-N <strong>force</strong> at O. <strong>The</strong> moment arm to the second <strong>force</strong> would <strong>be</strong> M/F 69.3/400 0.1732 m, which is 0.2 sin 60, thus determining the line <strong>of</strong> action<strong>of</strong> the single resultant <strong>force</strong> <strong>of</strong> 400 N.400 N 400 N≡ ≡O O O400 N 400 N 400 N69.3 N.m


c02.qxd 10/29/07 1:38 PM Page 53Article 2/5 Problems 53PROBLEMSIntroductory Problems2/57 <strong>The</strong> caster unit is subjected to the pair <strong>of</strong> 400-N<strong>force</strong>s shown. Determine the moment associatedwith these <strong>force</strong>s.Ans. M 14 N m CW35mm2/59 <strong>The</strong> top view <strong>of</strong> a revolving entrance door is shown.Two persons simultaneously approach the door andexert <strong>force</strong> <strong>of</strong> equal magnitudes as shown. If the resultingmoment about the door pivot axis at O is 25N m, determine the <strong>force</strong> magnitude F.Ans. F 16.18 NF15°0.8 m400 NO0.8 m15°– FProblem 2/59400 NProblem 2/572/58 A <strong>force</strong> F 60 N acts along the line AB. Determinethe equivalent <strong>force</strong>–couple system at point C.2/60 <strong>The</strong> indicated <strong>force</strong>–couple system is applied to asmall shaft at the center <strong>of</strong> the plate. Replace thissystem <strong>by</strong> a single <strong>force</strong> and specify the coordinate <strong>of</strong>the point on the x-axis through which the line <strong>of</strong> action<strong>of</strong> this resultalt <strong>force</strong> passes.M = 400 N·my10 mmBFyxOF = 6 kNxCAProblem 2/60Problem 2/58


c02.qxd 10/29/07 1:38 PM Page 5454 Chapter 2 Force Systems2/61 <strong>The</strong> bracket is spot welded to the end <strong>of</strong> the shaft atpoint O. To show the effect <strong>of</strong> the 900-N <strong>force</strong> on theweld, replace the <strong>force</strong> <strong>by</strong> its equivalent <strong>of</strong> a <strong>force</strong>and couple M at O. Express M in vector notation.Ans. M 90i N m2/63 Replace the 10-kN <strong>force</strong> acting on the steel column<strong>by</strong> an equivalent <strong>force</strong>–couple system at point O.This replacement is frequently done in the design <strong>of</strong>structures.Ans. R 10 kN, M O 0.75 kN m CCWy10 kN75 mmxO100 mmz1 m900 NOProblem 2/612/62 As part <strong>of</strong> a test, the two aircraft engines are revvedup and the propeller pitches are adjusted so as to resultin the fore and aft thrusts shown. What <strong>force</strong> F<strong>must</strong> <strong>be</strong> exerted <strong>by</strong> the ground on each <strong>of</strong> the mainbraked wheels at A and B to counteract the turningeffect <strong>of</strong> the two propeller thrusts? Neglect any effects<strong>of</strong> the nose wheel C, which is turned 90 andunbraked.Problem 2/632/64 Each propeller <strong>of</strong> the twin-screw ship develops a fullspeedthrust <strong>of</strong> 300 kN. In maneuvering the ship,one propeller is turning full speed ahead and theother full speed in reverse. What thrust P <strong>must</strong> eachtug exert on the ship to counteract the effect <strong>of</strong> theship’s propellers?50 m2 kNFF12 mAC2 kNB3 m 5 m120 mProblem 2/64Problem 2/62


c02.qxd 10/29/07 1:38 PM Page 55Article 2/5 Problems 55Representative Problems2/65 A lug wrench is used to tighten a square-head bolt. If250-N <strong>force</strong>s are applied to the wrench as shown, determinethe magnitude F <strong>of</strong> the equal <strong>force</strong>s exertedon the four contact points on the 25-mm bolt head sothat their external effect on the bolt is equivalent tothat <strong>of</strong> the two 250-N <strong>force</strong>s. Assume that the <strong>force</strong>sare perpendicular to the flats <strong>of</strong> the bolt head.Ans. F 3500 N25 mm2/67 <strong>The</strong> 180-N <strong>force</strong> is applied to the end <strong>of</strong> body OAB. If 50, determine the equivalent <strong>force</strong>–couple systemat the shaft axis O.Ans. F 169.1i 61.6j N, M O 41.9 N m CCWF = 180 NθBy30°120 mmAx150 mm250 NA350 mmC350 mmProblem 2/652/66 During a steady right turn, a person exerts the <strong>force</strong>sshown on the steering wheel. Note that each <strong>force</strong>consists <strong>of</strong> a tangential component and a radiallyinwardcomponent. Determine the moment exertedabout the steering column at O.30°250 NB8 NView C Detail(clearances exaggerated)Problem 2/672/68 A <strong>force</strong> F <strong>of</strong> magnitude 50 N is exerted on the automobileparking-brake lever at the position x 250mm. Replace the <strong>force</strong> <strong>by</strong> an equivalent <strong>force</strong>–couplesystem at the pivot point O.O100 mm15°xFO20°10°15°AOProblem 2/68B15°30°8 N375 mmProblem 2/66


c02.qxd 10/29/07 1:38 PM Page 5656 Chapter 2 Force Systems2/69 <strong>The</strong> tie-rod AB exerts the 250-N <strong>force</strong> on the steeringknuckle AO as shown. Replace this <strong>force</strong> <strong>by</strong> anequivalent <strong>force</strong>–couple system at O.Ans. F 43.4i 246j N, M O 60.0 N m CW10° F = 250 N235 mmy2/71 <strong>The</strong> system consisting <strong>of</strong> the bar OA, two identicalpulleys, and a section <strong>of</strong> thin tape is subjected to thetwo 180-N tensile <strong>force</strong>s shown in the figure. Determinethe equivalent <strong>force</strong>–couple system at point O.Ans. M 21.7 N m CCW180 NArBxr = 25 mmA50 mmr100 mmOO45°180 N50 mmProblem 2/692/70 <strong>The</strong> combined drive wheels <strong>of</strong> a front-wheel-driveautomobile are acted on <strong>by</strong> a 7000-N normal reaction<strong>force</strong> and a friction <strong>force</strong> F, both <strong>of</strong> which areexerted <strong>by</strong> the road surface. If it is known that theresultant <strong>of</strong> these two <strong>force</strong>s makes a 15 angle withthe vertical, determine the equivalent <strong>force</strong>–couplesystem at the car mass center G. Treat this as a twodimensionalproblem.500 mmGProblem 2/712/72 Calculate the moment M B <strong>of</strong> the 900-N <strong>force</strong> aboutthe bolt at B. Simplify your work <strong>by</strong> first replacingthe <strong>force</strong> <strong>by</strong> its equivalent <strong>force</strong>–couple system at A.200 mmCAB30°600 mm900 NA F1000mm7000 NProblem 2/70B40°Problem 2/72


c02.qxd 10/29/07 1:38 PM Page 57Article 2/5 Problems 572/73 <strong>The</strong> bracket is fastened to the girder <strong>by</strong> means <strong>of</strong> thetwo rivets A and B and supports the 2-kN <strong>force</strong>. Replacethis <strong>force</strong> <strong>by</strong> a <strong>force</strong> acting along the centerline<strong>be</strong>tween the rivets and a couple. <strong>The</strong>n redistributethis <strong>force</strong> and couple <strong>by</strong> replacing it <strong>by</strong> two <strong>force</strong>s,one at A and the other at B, and ascertain the <strong>force</strong>ssupported <strong>by</strong> the rivets.Ans. F A 0.8 kNF B 2.8 kN2/75 <strong>The</strong> weld at O can support a maximum <strong>of</strong> 2500 N <strong>of</strong><strong>force</strong> along each <strong>of</strong> the n- and t-directions and a maximum<strong>of</strong> 1400 N m <strong>of</strong> moment. Determine the allowablerange for the direction <strong>of</strong> the 2700-N <strong>force</strong>applied at A. <strong>The</strong> angle is restricted to 0 90.Ans. 22.2 47.8O0.7 mAθtA250 mmnProblem 2/752700 NxB300 mmProblem 2/73100 mm2 kN2/74 <strong>The</strong> angle plate is subjected to the two 250-N <strong>force</strong>sshown. It is desired to replace these <strong>force</strong>s <strong>by</strong> anequivalent set consisting <strong>of</strong> the 200-N <strong>force</strong> appliedat A and a second <strong>force</strong> applied at B. Determine they-coordinate <strong>of</strong> B.250 N200mmAxA200 N30°2/76 <strong>The</strong> device shown is a part <strong>of</strong> an automobile seatback-releasemechanism. <strong>The</strong> part is subjected to the4-N <strong>force</strong> exerted at A and a 300-N mm restoringmoment exerted <strong>by</strong> a hidden torsional spring. Determinethe y-intercept <strong>of</strong> the line <strong>of</strong> action <strong>of</strong> the singleequivalent <strong>force</strong>.10 mm40 mm300 N·mmyOAx15°F = 4 N240 mmProblem 2/76250 NB160 mmyyProblem 2/74


c02.qxd 10/29/07 1:38 PM Page 5858 Chapter 2 Force Systems2/6 RESULTANTSF 1F 2F 1RF 2R 1R 1F 3<strong>The</strong> <strong>properties</strong> <strong>of</strong> <strong>force</strong>, moment, and couple were developed in theprevious four articles. Now we are ready to descri<strong>be</strong> the resultant action <strong>of</strong>a group or system <strong>of</strong> <strong>force</strong>s. Most problems in mechanics deal with a system<strong>of</strong> <strong>force</strong>s, and it is usually necessary to reduce the system to its simplestform to descri<strong>be</strong> its action. <strong>The</strong> resultant <strong>of</strong> a system <strong>of</strong> <strong>force</strong>s is the simplest<strong>force</strong> combination which can replace the original <strong>force</strong>s without alteringthe external effect on the rigid body to which the <strong>force</strong>s are applied.Equilibrium <strong>of</strong> a body is the condition in which the resultant <strong>of</strong> all<strong>force</strong>s acting on the body is zero. This condition is studied in statics. Whenthe resultant <strong>of</strong> all <strong>force</strong>s on a body is not zero, the acceleration <strong>of</strong> the bodyis obtained <strong>by</strong> equating the <strong>force</strong> resultant to the product <strong>of</strong> the mass andacceleration <strong>of</strong> the body. This condition is studied in dynamics. Thus, thedetermination <strong>of</strong> resultants is basic to both statics and dynamics.<strong>The</strong> most common type <strong>of</strong> <strong>force</strong> system occurs when the <strong>force</strong>s allact in a single plane, say, the x-y plane, as illustrated <strong>by</strong> the system <strong>of</strong>three <strong>force</strong>s F 1 , F 2 , and F 3 in Fig. 2/13a. We obtain the magnitude anddirection <strong>of</strong> the resultant <strong>force</strong> R <strong>by</strong> forming the <strong>force</strong> polygon shown inpart b <strong>of</strong> the figure, where the <strong>force</strong>s are added head-to-tail in any sequence.Thus, for any system <strong>of</strong> coplanar <strong>force</strong>s we may write(a)yF FF 2y 2 F 33yF 1R F y 1yRθF 1xF 2x F 3xR x(b)Figure 2/13F 3xR F 1 F 2 F 3 ΣFR x ΣF x R y ΣF y R (ΣF x ) 2 (ΣF y ) 2 tan 1 R yR x tan 1 ΣF yΣF x(2/9)Graphically, the correct line <strong>of</strong> action <strong>of</strong> R may <strong>be</strong> obtained <strong>by</strong> preservingthe correct lines <strong>of</strong> action <strong>of</strong> the <strong>force</strong>s and adding them <strong>by</strong> theparallelogram law. We see this in part a <strong>of</strong> the figure for the case <strong>of</strong>three <strong>force</strong>s where the sum R 1 <strong>of</strong> F 2 and F 3 is added to F 1 to obtain R.<strong>The</strong> principle <strong>of</strong> transmissibility has <strong>be</strong>en used in this process.Algebraic MethodWe can use algebra to obtain the resultant <strong>force</strong> and its line <strong>of</strong> actionas follows:1. Choose a convenient reference point and move all <strong>force</strong>s to thatpoint. This process is depicted for a three-<strong>force</strong> system in Figs.2/14a and b, where M 1 , M 2 , and M 3 are the couples resulting fromthe transfer <strong>of</strong> <strong>force</strong>s F 1 , F 2 , and F 3 from their respective originallines <strong>of</strong> action to lines <strong>of</strong> action through point O.2. Add all <strong>force</strong>s at O to form the resultant <strong>force</strong> R, and add all couplesto form the resultant couple M O . We now have the single <strong>force</strong>–couple system, as shown in Fig. 2/14c.3. In Fig. 2/14d, find the line <strong>of</strong> action <strong>of</strong> R <strong>by</strong> requiring R to have amoment <strong>of</strong> M O about point O. Note that the <strong>force</strong> <strong>systems</strong> <strong>of</strong> Figs.2/14a and 2/14d are equivalent, and that Σ(Fd) in Fig. 2/14a is equalto Rd in Fig. 2/14d.


c02.qxd 10/29/07 1:38 PM Page 59Article 2/6 Resultants 59F 1F 2M 1 = F 1 d 1F 1d1 d 3O d 2F 2OF 3M 2 = F 2 d 2M 3 = F 3 d 3(a)F 3(b)M O = Σ(Fd)OR = ΣFOdRd = –— M OR(c)(d)Figure 2/14Principle <strong>of</strong> MomentsThis process is summarized in equation form <strong>by</strong>R ΣFM O ΣM Σ(Fd)(2/10)Rd M O<strong>The</strong> first two <strong>of</strong> Eqs. 2/10 reduce a given system <strong>of</strong> <strong>force</strong>s to a <strong>force</strong>–couple system at an arbitrarily chosen but convenient point O. <strong>The</strong> lastequation specifies the distance d from point O to the line <strong>of</strong> action <strong>of</strong> R,and states that the moment <strong>of</strong> the resultant <strong>force</strong> about any point Oequals the sum <strong>of</strong> the moments <strong>of</strong> the original <strong>force</strong>s <strong>of</strong> the system aboutthe same point. This extends Varignon’s theorem to the case <strong>of</strong> nonconcurrent<strong>force</strong> <strong>systems</strong>; we call this extension the principle <strong>of</strong> moments.For a concurrent system <strong>of</strong> <strong>force</strong>s where the lines <strong>of</strong> action <strong>of</strong> all<strong>force</strong>s pass through a common point O, the moment sum ΣM O aboutthat point is zero. Thus, the line <strong>of</strong> action <strong>of</strong> the resultant R ΣF, determined<strong>by</strong> the first <strong>of</strong> Eqs. 2/10, passes through point O. For a parallel<strong>force</strong> system, select a coordinate axis in the direction <strong>of</strong> the <strong>force</strong>s.If the resultant <strong>force</strong> R for a given <strong>force</strong> system is zero, the resultant<strong>of</strong> the system need not <strong>be</strong> zepo <strong>be</strong>cause the resultant may <strong>be</strong> a couple.<strong>The</strong> three <strong>force</strong>s in Fig. 2/15, for instance, have a zero resultant <strong>force</strong>but have a resultant clockwise couple M F 3 d.F 1F 1 + F 2 = –F 3F 1F 2F 2dF 3Figure 2/15


c02.qxd 10/29/07 1:38 PM Page 6060 Chapter 2 Force SystemsSample Problem 2/9Determine the resultant <strong>of</strong> the four <strong>force</strong>s and one couple which act on theplate shown.60 N2 m50 N5 mySolution. Point O is selected as a convenient reference point for the <strong>force</strong>–couplesystem which is to represent the given system.[R x ΣF x ][R y ΣF y ][R R x2 R y 2 ] tan1 R yR x[M O Σ(Fd)]R x 40 80 cos 30 60 cos 45 66.9 NR y 50 80 sin 30 60 cos 45 132.4 NR (66.9) 2 (132.4) 2 148.3 N tan1 132.466.9 63.2M O 140 50(5) 60 cos 45(4) 60 sin 45(7) 237 N mAns.Ans.<strong>The</strong> <strong>force</strong>–couple system consisting <strong>of</strong> R and M O is shown in Fig. a.We now determine the final line <strong>of</strong> action <strong>of</strong> R such that R alone representsthe original system.[Rd M O ]148.3d 237 d 1.600 mAns.Hence, the resultant R may <strong>be</strong> applied at any point on the line which makes a63.2 angle with the x-axis and is tangent at point A to a circle <strong>of</strong> 1.600-m radiuswith center O, as shown in part b <strong>of</strong> the figure. We apply the equation Rd M O inan absolute-value sense (ignoring any sign <strong>of</strong> M O ) and let the physics <strong>of</strong> the situation,as depicted in Fig. a, dictate the final placement <strong>of</strong> R. Had M O <strong>be</strong>en counterclockwise,the correct line <strong>of</strong> action <strong>of</strong> R would have <strong>be</strong>en the tangent at point B.<strong>The</strong> resultant R may also <strong>be</strong> located <strong>by</strong> determining its intercept distance bto point C on the x-axis, Fig. c. With R x and R y acting through point C, only R yexerts a moment about O so thatR y b M O and b 237132.4 1.792 mAlternatively, the y-intercept could have <strong>be</strong>en obtained <strong>by</strong> noting that the momentabout O would <strong>be</strong> due to R x only.A more formal approach in determining the final line <strong>of</strong> action <strong>of</strong> R is to usethe vector expressionr R M Owhere r xi yj is a position vector running from point O to any point on theline <strong>of</strong> action <strong>of</strong> R. Substituting the vector expressions for r, R, and M O and carryingout the cross product result in(xi yj) (66.9i 132.4j) 237k(132.4x 66.9y)k 237kThus, the desired line <strong>of</strong> action, Fig. c, is given <strong>by</strong>132.4x 66.9y 237By setting y 0, we obtain x 1.792 m, which agrees with our earlier calculation<strong>of</strong> the distance b.45°140 N·m2 m80 N2 m40 N O 30° x1 m(a)(b)(c)|M O | =237 N·mOR = 148.3 N1.600 mA132.4x – 66.9y =–237RCOO<strong>by</strong>yR = 148.3 Nθ = 63.2°x63.2°Helpful Hints We note that the choice <strong>of</strong> point O asa moment center eliminates any momentsdue to the two <strong>force</strong>s whichpass through O. Had the clockwisesign convention <strong>be</strong>en adopted, M Owould have <strong>be</strong>en 237 N m, withthe plus sign indicating a sensewhich agrees with the sign convention.Either sign convention, <strong>of</strong>course, leads to the conclusion <strong>of</strong> aclockwise moment M O . Note that the vector approachyields sign information automatically,whereas the scalar approachis more physically oriented. Youshould master both methods.Bxx


c02.qxd 10/29/07 1:38 PM Page 61Article 2/6 Problems 61PROBLEMSIntroductory Problems2/77 Calculate the magnitude <strong>of</strong> the tension T and theangle for which the eye bolt will <strong>be</strong> under a resultantdownward <strong>force</strong> <strong>of</strong> 15 kN.Ans. T 12.85 kN, 38.92/79 Determine the equivalent <strong>force</strong>–couple system at theorigin O for each <strong>of</strong> the three cases <strong>of</strong> <strong>force</strong>s <strong>be</strong>ingapplied to the edge <strong>of</strong> a circular disk. If the resultantcan <strong>be</strong> so expressed, replace this <strong>force</strong>–couple systemwith a stand-alone <strong>force</strong>.Ans. (a) R 2Fj along x r(b) R Fi along y 3r(c) R Fi along y rFyFyFFFyxxr Or OrOxxTθ30°6 kNF(a)FFF(b)Problem 2/79(c)FProblem 2/772/78 Determine the resultant R <strong>of</strong> the four <strong>force</strong>s actingon the gusset plate. Also find the magnitude <strong>of</strong> Rand the angle x which the resultant makes with thex-axis.30 kN20°50 kN20°y8 kN60 kN2/80 Determine the height h above the base B at whichthe resultant <strong>of</strong> the three <strong>force</strong>s acts.600 mm650 N600 mm600 mmProblem 2/80300 N250 N2/81 Where does the resultant <strong>of</strong> the two <strong>force</strong>s act?Ans. 10.70 m to the left <strong>of</strong> AB40°680 Nx300mm500mmA40 kNProblem 2/78660 NProblem 2/81


c02.qxd 10/29/07 1:38 PM Page 6262 Chapter 2 Force Systems2/82 Under nonuniform and slippery road conditions, thetwo <strong>force</strong>s shown are exerted on the two rear-drivewheels <strong>of</strong> the pickup truck, which has a limited-sliprear differential. Determine the y-intercept <strong>of</strong> the resultant<strong>of</strong> this <strong>force</strong> system.y2/84 Determine the magnitude <strong>of</strong> the <strong>force</strong> F applied tothe handle which will make the resultant <strong>of</strong> thethree <strong>force</strong>s pass through O.F250 N750 mmGx300 mm750 mm500 NOProblem 2/822/83 If the resultant <strong>of</strong> the two <strong>force</strong>s and couple Mpasses through point O, determine M.Ans. M 148.0 N m CCWMO1675 mm150mm1125 mm150mm30°60°320 NProblem 2/842/85 Determine and locate the resultant R <strong>of</strong> the tw<strong>of</strong>orces and one couple acting on the I-<strong>be</strong>am.Ans. R 4 kN down at x 5 mx150 100mm120 Nmm240 N52 kN·m3 m 3 m 3 m10 kN160mm400 NProblem 2/836 kNProblem 2/85


c02.qxd 10/29/07 1:38 PM Page 63Article 2/6 Problems 632/86 A commercial airliner with four jet engines, eachproducing 90 kN <strong>of</strong> forward thrust, is in a steady,level cruise when engine num<strong>be</strong>r 3 suddenly fails.Determine and locate the resultant <strong>of</strong> the three remainingengine thrust vectors. Treat this as a twodimensionalproblem.90 kN349 m12 mRepresentative Problems2/88 <strong>The</strong> directions <strong>of</strong> the two thrust vectors <strong>of</strong> an experimentalaircraft can <strong>be</strong> independently changed fromthe conventional forward direction within limits. Forthe thrust configuration shown, determine theequivalent <strong>force</strong>–couple system at point O. <strong>The</strong>n replacethis <strong>force</strong>–couple system <strong>by</strong> a single <strong>force</strong> andspecify the point on the x-axis through which theline <strong>of</strong> action <strong>of</strong> this resultant passes. <strong>The</strong>se resultsare vital to assessing design performance.10 my90 kN90 kN2112 m9 mT3 m3 m15°TABOxProblem 2/862/87 Replace the three <strong>force</strong>s acting on the <strong>be</strong>nt pipe <strong>by</strong> asingle equivalent <strong>force</strong> R. Specify the distance xfrom point O to the point on the x-axis throughwhich the line <strong>of</strong> action <strong>of</strong> R passes.Ans. R 200i 80j N, x 1.625 m (<strong>of</strong>f pipe)yO160 N200 N250 mmxProblem 2/882/89 Determine the resultant R <strong>of</strong> the three <strong>force</strong>s actingon the simple truss. Specify the points on the x- andy-axes through which R <strong>must</strong> pass.Ans. R 15i 47.3j kNx 7.42 m, y 23.4 my25 kN5 m20 kN30°250 mm250 mm240 N125 mmO3 m 6 m 3 mxProblem 2/8730 kNProblem 2/89


c02.qxd 10/29/07 1:38 PM Page 6464 Chapter 2 Force Systems2/90 <strong>The</strong> gear and attached V-<strong>be</strong>lt pulley are turningcounterclockwise and are subjected to the tooth load<strong>of</strong> 1600 N and the 800-N and 450-N tensions in theV-<strong>be</strong>lt. Represent the action <strong>of</strong> these three <strong>force</strong>s <strong>by</strong>a resultant <strong>force</strong> R at O and a couple <strong>of</strong> magnitudeM. Is the unit slowing down or speeding up?Problem 2/902/91 <strong>The</strong> design specifications for the attachment at A forthis <strong>be</strong>am depend on the magnitude and location <strong>of</strong>the applied loads. Represent the resultant <strong>of</strong> thethree <strong>force</strong>s and couple <strong>by</strong> a single <strong>force</strong> R at A and acouple M. Specify the magnitude <strong>of</strong> R.Ans. R 1.879i 1.684j kN, R 2.52 kNM 14.85 kN m CWy450 N800 N1.8 mx15°15°4 kN280mm 150mm1.2 mOy20°20°30°1600 NProblem 2/922/93 Two integral pulleys are subjected to the <strong>be</strong>lt tensionsshown. If the resultant R <strong>of</strong> these <strong>force</strong>s passesthrough the center O, determine T and the magnitude<strong>of</strong> R and the counterclockwise angle it makeswith the x-axis.Ans. T 60 N, R 193.7 N, 34.6T80 N120 N30°200 mm160 N200 mm30°100mmOyOθP250 mm20°200 N200 mmx150 N1.5 mA0.8 m3 kNxProblem 2/918 kN·m2 kN2/92 In the equilibrium position shown, the resultant <strong>of</strong>the three <strong>force</strong>s acting on the <strong>be</strong>ll crank passesthrough the <strong>be</strong>aring O. Determine the vertical <strong>force</strong>P. Does the result depend on ?Problem 2/932/94 While sliding a desk toward the doorway, three studentsexert the <strong>force</strong>s shown in the overhead view.Determine the equivalent <strong>force</strong>–couple system atpoint A. <strong>The</strong>n determine the equation <strong>of</strong> the line <strong>of</strong>action <strong>of</strong> the resultant <strong>force</strong>.100 Ny1.5 m60 N80 NA0.75 mxProblem 2/94


c02.qxd 10/29/07 1:38 PM Page 65Article 2/6 Problems 652/95 Under nonuniform and slippery road conditions, thefour <strong>force</strong>s shown are exerted on the four drivewheels <strong>of</strong> the all-wheel-drive vehicle. Determine theresultant <strong>of</strong> this system and the x- and y-intercepts<strong>of</strong> its line <strong>of</strong> action. Note that the front and reartracks are equal (i.e., AB CD).Ans. R 903i 175j N, (0, 0.253) m, (1.308, 0) m300 N300 NProblem 2/952/96 <strong>The</strong> rolling rear wheel <strong>of</strong> a front-wheel-drive automobilewhich is accelerating to the right is subjectedto the five <strong>force</strong>s and one moment shown. <strong>The</strong> <strong>force</strong>sA x 240 N and A y 2000 N are <strong>force</strong>s transmittedfrom the axle to the wheel, F 160 N is the friction<strong>force</strong> exerted <strong>by</strong> the road surface on the tire, N 2400 N is the normal reaction <strong>force</strong> exerted <strong>by</strong> theroad surface, and W 400 N is the weight <strong>of</strong> thewheel/tire unit. <strong>The</strong> couple M 3 N m is the <strong>be</strong>aringfriction moment. Determine and locate the resultant<strong>of</strong> the system.yBAxMA200 N1520 mm 1120 mmyA yNCDA x30°750 mm760 mm760 mm2/97 A rear-wheel-drive car is stuck in the snow <strong>be</strong>tweenother parked cars as shown. In an attempt to freethe car, three students exert <strong>force</strong>s on the car atpoints A, B, and C while the driver’s actions result ina forward thrust <strong>of</strong> 200 N acting parallel to the plane<strong>of</strong> rotation <strong>of</strong> each rear wheel. Treating the problemas two-dimensional, determine the equivalent<strong>force</strong>–couple system at the car center <strong>of</strong> mass G andlocate the position x <strong>of</strong> the point on the car centerlinethrough which the resultant passes. Neglect all<strong>force</strong>s not shown.Ans. R 925i 567j NM G 690 N m CCWx 1.218 m200 N825mm400 N 825mm200 NProblem 2/972/98 An exhaust system for a pickup truck is shown inthe figure. <strong>The</strong> weights W h , W m , and W t <strong>of</strong> the headpipe,muffler, and tailpipe are 10, 100, and 50 N, respectively,and act at the indicated points. If theexhaust-pipe hanger at point A is adjusted so that itstension F A is 50 N, determine the required <strong>force</strong>s inthe hangers at points B, C, and D so that the<strong>force</strong>–couple system at point O is zero. Why is a zer<strong>of</strong>orce–couple system at O desirable?F AAG250 N 30°F ByBF Cx30°C1650mm350 N1800mm30°F DWNFOABW h W mW tCDr = 0.350.5 0.2 0.65 0.650.5 0.4Problem 2/96Dimensions in metersProblem 2/98


c02.qxd 10/29/07 1:38 PM Page 66⎭66 Chapter 2 Force SystemsSECTION BTHREE-DIMENSIONAL FORCE SYSTEMSz2/7 RECTANGULAR C OMPONENTSMany problems in mechanics require analysis in three dimensions,and for such problems it is <strong>of</strong>ten necessary to resolve a <strong>force</strong> into itsthree mutually perpendicular components. <strong>The</strong> <strong>force</strong> F acting at point Oin Fig. 2/16 has the rectangular components F x , F y , F z , whereF z kFyF x F cos x F F2x F2y F2zF y F cos yF F x i F y j F z k(2/11)θ zθ yF z F cos z F F(i cos x j cos y k cos z )kiOjF x iθ xF y jx<strong>The</strong> unit vectors i, j, and k are in the x-, y-, and z-directions, respectively.Using the direction cosines <strong>of</strong> F, which are l cos x , m cos y ,and n cos z , where l 2 m 2 n 2 1, we may write the <strong>force</strong> asFigure 2/16F F(li mj nk)(2/12)We may regard the right-side expression <strong>of</strong> Eq. 2/12 as the <strong>force</strong>magnitude F times a unit vector n F which characterizes the direction <strong>of</strong>F, orF Fn F(2/12a)zFA (x 1 , y 1 , z 1 )⎫⎪⎪⎪⎬⎪⎪⎪⎭B (x 2 , y 2 , z 2 )(z 2 – z 1 )It is clear from Eqs. 2/12 and 2/12a that n F li mj nk, whichshows that the scalar components <strong>of</strong> the unit vector n F are the directioncosines <strong>of</strong> the line <strong>of</strong> action <strong>of</strong> F.In solving three-dimensional problems, one <strong>must</strong> usually find the x,y, and z scalar components <strong>of</strong> a <strong>force</strong>. In most cases, the direction <strong>of</strong> a<strong>force</strong> is descri<strong>be</strong>d (a) <strong>by</strong> two points on the line <strong>of</strong> action <strong>of</strong> the <strong>force</strong> or(b) <strong>by</strong> two angles which orient the line <strong>of</strong> action.(a) Specification <strong>by</strong> two points on the line <strong>of</strong> action <strong>of</strong> the <strong>force</strong>.If the coordinates <strong>of</strong> points A and B <strong>of</strong> Fig. 2/17 are known, the <strong>force</strong> Fmay <strong>be</strong> written asx⎫⎪(y 2 – y 1 )⎪⎪⎬⎪⎪⎪Figure 2/17⎫ ⎬⎭(x 2 – x 1 )yF Fn F F ABlAB F (x 2 x 1 )i (y 2 y 1 )j (z 2 z 1 )k(x 2 x 1 ) 2 (y 2 y 1 ) 2 (z 2 z 1 ) 2Thus the x, y, and z scalar components <strong>of</strong> F are the scalar coefficients <strong>of</strong>the unit vectors i, j, and k, respectively.


c02.qxd 10/29/07 1:38 PM Page 67Article 2/7 Rectangular Components 67(b) Specification <strong>by</strong> two angles which orient the line <strong>of</strong> action <strong>of</strong>the <strong>force</strong>. Consider the geometry <strong>of</strong> Fig. 2/18. We assume that the angles and are known. First resolve F into horizontal and vertical components.zF zθF xy F cos F z F sin <strong>The</strong>n resolve the horizontal component F xy into x- and y-components.F x F xy cos F cos cos F y F xy sin F cos sin <strong>The</strong> quantities F x , F y , and F z are the desired scalar components <strong>of</strong> F.<strong>The</strong> choice <strong>of</strong> orientation <strong>of</strong> the coordinate system is arbitrary, withconvenience <strong>be</strong>ing the primary consideration. However, we <strong>must</strong> use aright-handed set <strong>of</strong> axes in our three-dimensional work to <strong>be</strong> consistentwith the right-hand-rule definition <strong>of</strong> the cross product. When we rotatefrom the x- to the y-axis through the 90 angle, the positive direction forthe z-axis in a right-handed system is that <strong>of</strong> the advancement <strong>of</strong> aright-handed screw rotated in the same sense. This is equivalent to theright-hand rule.xφFF yF xF xyyFigure 2/18Dot ProductWe can express the rectangular components <strong>of</strong> a <strong>force</strong> F (or anyother vector) with the aid <strong>of</strong> the vector operation known as the dot orscalar product (see item 6 in Art. C/7 <strong>of</strong> Appendix C). <strong>The</strong> dot product <strong>of</strong>two vectors P and Q, Fig. 2/19a, is defined as the product <strong>of</strong> their magnitudestimes the cosine <strong>of</strong> the angle <strong>be</strong>tween them. It is written asP Q PQ cos We can view this product either as the orthogonal projection P cos <strong>of</strong>P in the direction <strong>of</strong> Q multiplied <strong>by</strong> Q, or as the orthogonal projectionQ cos <strong>of</strong> Q in the direction <strong>of</strong> P multiplied <strong>by</strong> P. In either case the dotproduct <strong>of</strong> the two vectors is a scalar quantity. Thus, for instance, wecan express the scalar component F x F cos x <strong>of</strong> the <strong>force</strong> F in Fig.2/16 as F x F i, where i is the unit vector in the x-direction.PFαQF n = F·nnF n = F·nn (unit vector)(a)(b)Figure 2/19


c02.qxd 10/29/07 1:38 PM Page 6868 Chapter 2 Force SystemsIn more general terms, if n is a unit vector in a specified direction,the projection <strong>of</strong> F in the n-direction, Fig. 2/19b, has the magnitudeF n F n. If we want to express the projection in the n-direction as avector quantity, then we multiply its scalar component, expressed <strong>by</strong>F n, <strong>by</strong> the unit vector n to give F n (F n)n. We may write this asF n F nn without ambiguity <strong>be</strong>cause the term nn is not defined, andso the complete expression cannot <strong>be</strong> misinterpreted as F (nn).If the direction cosines <strong>of</strong> n are , , and , then we may write n invector component form like any other vector asn i j kwhere in this case its magnitude is unity. If the direction cosines <strong>of</strong> Fwith respect to reference axes x-y-z are l, m, and n, then the projection<strong>of</strong> F in the n-direction <strong>be</strong>comesF n F n F(li mj nk) (i j k) F(l m n)<strong>be</strong>causeandi i j j k k 1i j j i i k k i j k k j 0<strong>The</strong> latter two sets <strong>of</strong> equations are true <strong>be</strong>cause i, j, and k have unitlength and are mutually perpendicular.Angle <strong>be</strong>tween Two VectorsIf the angle <strong>be</strong>tween the <strong>force</strong> F and the direction specified <strong>by</strong> theunit vector n is , then from the dot-product definition we have F n Fn cos F cos , where n n 1. Thus, the angle <strong>be</strong>tween F and nis given <strong>by</strong> cos 1 F nF(2/13)PIn general, the angle <strong>be</strong>tween any two vectors P and Q is cos 1 P QPQ(2/13a)P′αQFigure 2/20If a <strong>force</strong> F is perpendicular to a line whose direction is specified <strong>by</strong> theunit vector n, then cos 0, and F n 0. Note that this relationshipdoes not mean that either F or n is zero, as would <strong>be</strong> the case withscalar multiplication where (A)(B) 0 requires that either A or B (orboth) <strong>be</strong> zero.<strong>The</strong> dot-product relationship applies to nonintersecting vectors aswell as to intersecting vectors. Thus, the dot product <strong>of</strong> the nonintersectingvectors P and Q in Fig. 2/20 is Q times the projection <strong>of</strong> P on Q,or PQ cos PQ cos <strong>be</strong>cause P and P are the same when treated asfree vectors.


c02.qxd 10/29/07 1:38 PM Page 69Article 2/7 Rectangular Components 69Sample Problem 2/10A <strong>force</strong> F with a magnitude <strong>of</strong> 100 N is applied at the origin O <strong>of</strong> the axesx-y-z as shown. <strong>The</strong> line <strong>of</strong> action <strong>of</strong> F passes through a point A whose coordinatesare 3 m, 4 m, and 5 m. Determine (a) the x, y, and z scalar components <strong>of</strong>F, (b) the projection F xy <strong>of</strong> F on the x-y plane, and (c) the projection F OB <strong>of</strong> Falong the line OB.Solution. Part (a). We <strong>be</strong>gin <strong>by</strong> writing the <strong>force</strong> vector F as its magnitudeF times a unit vector n OA .F Fn OA F OAlOA 100 3i 4j 5k 3 2 4 2 5 2 100[0.424i 0.566j 0.707k] 42.4i 56.6j 70.7k Nz5 mOz4 m3 mF = 100 NAy6 m6 mxFB2 m<strong>The</strong> desired scalar components are thusF zF x 42.4 N F y 56.6 N F z 70.7 NAns.yPart (b).<strong>The</strong> cosine <strong>of</strong> the angle xy <strong>be</strong>tween F and the x-y plane isF y3 2 4 2cos xy 3 2 4 2 5 0.7072so that F xy F cos xy 100(0.707) 70.7 NAns.OF xθxyF xy = 70.7 NPart (c).<strong>The</strong> unit vector n OB along OB isxn OB OBlOB6i 6j 2k 0.688i 0.688j 0.229k6 2 6 2 2 2F<strong>The</strong> scalar projection <strong>of</strong> F on OB isF OB F n OB (42.4i 56.6j 70.7k) (0.688i 0.688j 0.229k) (42.4)(0.688) (56.6)(0.688) (70.7)(0.229)z 84.4NAns.yn OBIf we wish to express the projection as a vector, we writeF OB = 84.4 NF OB F n OB n OB 84.4(0.688i 0.688j 0.229k) 58.1i 58.1j 19.35k NOHelpful Hintsx In this example all scalar componentsare positive. Be prepared for the casewhere a direction cosine, and hencethe scalar component, are negative. <strong>The</strong> dot product automatically findsthe projection or scalar component<strong>of</strong> F along line OB as shown.


c02.qxd 10/29/07 1:38 PM Page 7070 Chapter 2 Force SystemsPROBLEMSz, mmIntroductory ProblemsB (–25, 50, 40)2/99 Express F as a vector in terms <strong>of</strong> the unit vectorsi, j, and k. Determine the angle <strong>be</strong>tween F and they-axis.Ans. F 18.86i 23.6j 51.9k N y 113.1zx, mmOA (15, –20, –25)F = 750 Ny, mm110F = 60 N40O50 yxDimensions in millimetersProblem 2/99Problem 2/1012/102 Express F as a vector in terms <strong>of</strong> the unit vectors i,j, and k. Determine the projection, both as a scalarand as a vector, <strong>of</strong> F onto line OA, which lies in thex-y plane.z40°F = 5 kN2/100 <strong>The</strong> 70-m microwave transmission tower is steadied<strong>by</strong> three guy cables as shown. Cable AB carriesa tension <strong>of</strong> 12 kN. Express the corresponding <strong>force</strong>on point B as a vector.zxO30° 35°AyAProblem 2/10235 mB25 mx70 m10 mO20 m30 mCD20 m40 my2/103 <strong>The</strong> <strong>force</strong> F has a magnitude <strong>of</strong> 900 N and acts alongthe diagonal <strong>of</strong> the parallelepiped as shown. Express Fin terms <strong>of</strong> its magnitude times the appropriate unitvector and determine its x-, y-, and z-components.Ans. F 900 [ 1 3 i 2 3 j 2 k] 3 NF x = 300 N, F y = 600 N, F z = 600 NzProblem 2/1002/101 Express the <strong>force</strong> F as a vector in terms <strong>of</strong> theunit vectors i, j, and k. Determine the angles x , y , and z which F makes with the positive x-, y-,and z-axes.Ans. F 290i 507j 471k N x 112.7, y 47.5, z 51.14 m2 mF = 900 N4 myxProblem 2/103


c02.qxd 10/29/07 1:38 PM Page 71Article 2/7 Problems 712/104 <strong>The</strong> turnbuckle is tightened until the tension inthe cable AB equals 2.4 kN. Determine the vectorexpression for the tension T as a <strong>force</strong> acting onmem<strong>be</strong>r AD. Also find the magnitude <strong>of</strong> the projection<strong>of</strong> T along the line AC.z2/106 Use the result cited for Prob. 2/105 and determinethe magnitude T GF <strong>of</strong> the projection <strong>of</strong> Tonto line GF.Representative ProblemsDA2/107 <strong>The</strong> tension in the supporting cable AB is 10 kN.Write the <strong>force</strong> which the cable exerts on the boomBC as a vector T. Determine the angles x , y , and z which the line <strong>of</strong> action <strong>of</strong> T forms with the positivex-, y-, and z-axes.Ans. T 10[0.406i 0.761j 0.507k] kN x 66.1, y 139.5, z 59.55 mzxProblem 2/1042/105 <strong>The</strong> rigid pole and cross-arm assembly is supported<strong>by</strong> the three cables shown. A turnbuckle at D istightened until it induces a tension T in CD <strong>of</strong> 1.2kN. Express T as a vector. Does it make any differencein the result which coordinate system is used?Ans. T 0.321i 0.641j 0.962k kN, NoFxx′1 mC2 m3 mG1 mBAzOOB1.5 m1.5 m3 m3 mz′1.5 m2 mCE1 my′T = 1.2 kNDyyProblem 2/1072/108 <strong>The</strong> cable BC carries a tension <strong>of</strong> 750 N. Write thistension as a <strong>force</strong> T acting on point B in terms <strong>of</strong>the unit vectors i, j, and k. <strong>The</strong> elbow at A forms aright angle.zxAC0.7 m5 m4 m1.2 mCOyT = 10 kN5 m 2.5 mProblem 2/1081.6 mB30°AB0.8 myxProblem 2/105


c02.qxd 10/29/07 1:38 PM Page 7272 Chapter 2 Force Systems2/109 Derive the expression for the projection F DC <strong>of</strong> the<strong>force</strong> F onto the line directed from D to C.(b 2 a 2 )FAns. F DC a 2 b 2 a 2 b 2 c 2D2/112 <strong>The</strong> rectangular plate is supported <strong>by</strong> hinges alongits side BC and <strong>by</strong> the cable AE. If the cable tensionis 300 N, determine the projection onto line BC <strong>of</strong>the <strong>force</strong> exerted on the plate <strong>by</strong> the cable. Notethat E is the midpoint <strong>of</strong> the horizontal upper edge<strong>of</strong> the structural support.FBCAbCProblem 2/109acE25°T = 300 N400 mm1200 mmD2/110 If M is the center <strong>of</strong> the face ABCD <strong>of</strong> the rectangularsolid, express F as a vector in terms <strong>of</strong> the unitvectors i, j, and k. Determine the scalar projection<strong>of</strong> F along line AE. <strong>The</strong> quantities F, a, b, and c areknown.EzBBAProblem 2/1122/113 <strong>The</strong> access door is held in the 30 open position <strong>by</strong>the chain AB. If the tension in the chain is 100 N,determine the projection <strong>of</strong> the tension <strong>force</strong> ontothe diagonal axis CD <strong>of</strong> the door.Ans. T CD 46.0 NcOAMFBxbProblem 2/1102/111 Determine the angle <strong>be</strong>tween the 200-N <strong>force</strong> andline OC.Ans. 54.9DaCyC1200 mm30°A900mm900 mmDzOF = 200 NBProblem 2/11380 mmxA240 mmC120 mmyProblem 2/111


c02.qxd 10/29/07 1:38 PM Page 73Article 2/7 Problems 732/114 <strong>The</strong> spring <strong>of</strong> constant k 2.6 kN/m is attached tothe disk at point A and to the end fitting at point Bas shown. <strong>The</strong> spring is unstretched when A and B are both zero. If the disk is rotated 15 clockwiseand the end fitting is rotated 30 counterclockwise,determine a vector expression for the <strong>force</strong> F whichthe spring exerts at point A. <strong>The</strong> magnitude <strong>of</strong> thespring <strong>force</strong> is the constant k multiplied <strong>by</strong> the deflection(lengthening or shortening) <strong>of</strong> the spring.Ans. F 3.12i 41.1j 7.52k N2/116 A <strong>force</strong> F is applied to the surface <strong>of</strong> the sphere asshown. <strong>The</strong> angles and locate point P, and pointM is the midpoint <strong>of</strong> ON. Express F in vector form,using the given x-, y-, and z-coordinates.Ans.F F (2 sin 1)(i cos j sin ) k(2 cos )5 4 sin zFzxOA250 mmOφRPθB = 30°A = 15°θMyy900 mm k = 2.6 kN/m200 mmNθBxProblem 2/114Problem 2/1162/115 Determine the x-, y-, and z-components <strong>of</strong> <strong>force</strong> Fwhich acts on the tetrahedron as shown. <strong>The</strong> quantitiesa, b, c, and F are known, and M is the midpoint<strong>of</strong> edge AB.2acFAns. F x a 2 b 2 a 2 b 2 4c 2z2bcFF y a 2 b 2 a 2 b 2 4c 2F z F a 2 b 2a 2 b 2 4c 2CcObBFyaMAxProblem 2/115


c02.qxd 10/29/07 1:38 PM Page 7474 Chapter 2 Force Systems2/8 MOMENT AND C OUPLEIn two-dimensional analyses it is <strong>of</strong>ten convenient to determine amoment magnitude <strong>by</strong> scalar multiplication using the moment-armrule. In three dimensions, however, the determination <strong>of</strong> the perpendiculardistance <strong>be</strong>tween a point or line and the line <strong>of</strong> action <strong>of</strong> the <strong>force</strong>can <strong>be</strong> a tedious computation. A vector approach with cross-productmultiplication then <strong>be</strong>comes advantageous.Moments in Three DimensionsConsider a <strong>force</strong> F with a given line <strong>of</strong> action acting on a body, Fig.2/21a, and any point O not on this line. Point O and the line <strong>of</strong> F establisha plane A. <strong>The</strong> moment M O <strong>of</strong> F about an axis through O normal tothe plane has the magnitude M O Fd, where d is the perpendicular distancefrom O to the line <strong>of</strong> F. This moment is also referred to as the moment<strong>of</strong> F about the point O.<strong>The</strong> vector M O is normal to the plane and is directed along the axisthrough O. We can descri<strong>be</strong> both the magnitude and the direction <strong>of</strong> M O<strong>by</strong> the vector cross-product relation introduced in Art. 2/4. (Refer toitem 7 in Art. C/7 <strong>of</strong> Appendix C.) <strong>The</strong> vector r runs from O to any pointon the line <strong>of</strong> action <strong>of</strong> F. As descri<strong>be</strong>d in Art. 2/4, the cross product <strong>of</strong> rand F is written r F and has the magnitude (r sin )F, which is thesame as Fd, the magnitude <strong>of</strong> M O .<strong>The</strong> correct direction and sense <strong>of</strong> the moment are established <strong>by</strong>the right-hand rule, descri<strong>be</strong>d previously in Arts. 2/4 and 2/5. Thus,with r and F treated as free vectors emanating from O, Fig. 2/21b, thethumb points in the direction <strong>of</strong> M O if the fingers <strong>of</strong> the right hand curlin the direction <strong>of</strong> rotation from r to F through the angle . <strong>The</strong>refore,we may write the moment <strong>of</strong> F about the axis through O asAM OOF(a)M OdrFαAO(b)αrM O r F(2/14)<strong>The</strong> order r F <strong>of</strong> the vectors <strong>must</strong> <strong>be</strong> maintained <strong>be</strong>cause F rwould produce a vector with a sense opposite to that <strong>of</strong> M O ; that is,F r M O .Figure 2/21Evaluating the Cross Product<strong>The</strong> cross-product expression for M O may <strong>be</strong> written in the determinantformM O i r xF xjr yF ykzr zF(2/15)(Refer to item 7 in Art. C/7 <strong>of</strong> Appendix C if you are not already familiarwith the determinant representation <strong>of</strong> the cross product.) Note thesymmetry and order <strong>of</strong> the terms, and note that a right-handed coordinatesystem <strong>must</strong> <strong>be</strong> used. Expansion <strong>of</strong> the determinant givesM O (r y F z r z F y )i (r z F x r x F z )j (r x F y r y F x )k


c02.qxd 10/29/07 1:38 PM Page 75Article 2/8 Moment and Couple 75To gain more confidence in the cross-product relationship, examinethe three components <strong>of</strong> the moment <strong>of</strong> a <strong>force</strong> about a point asFF zobtained from Fig. 2/22. This figure shows the three components <strong>of</strong> a<strong>force</strong> F acting at a point A located relative to O <strong>by</strong> the vector r. <strong>The</strong>zscalar magnitudes <strong>of</strong> the moments <strong>of</strong> these <strong>force</strong>s about the positiveAF yx-, y-, and z-axes through O can <strong>be</strong> obtained from the moment-armM zrule, and arer y zF xrM x r y F z r z F y M y r z F x r x F z M z r x F y r y F xwhich agree with the respective terms in the determinant expansion forthe cross product r F.OMr yxr yM xxMoment about an Arbitrary AxisFigure 2/22We can now obtain an expression for the moment M <strong>of</strong> F about anyaxis through O, as shown in Fig. 2/23. If n is a unit vector in the-direction, then we can use the dot-product expression for the component<strong>of</strong> a vector as descri<strong>be</strong>d in Art. 2/7 to obtain M O n, the component<strong>of</strong> M O in the direction <strong>of</strong> . This scalar is the magnitude <strong>of</strong> the momentM <strong>of</strong> F about .To obtain the vector expression for the moment M <strong>of</strong> F about ,multiply the magnitude <strong>by</strong> the directional unit vector n to obtainM OλnM λrFM (r F n)n(2/16)Owhere r F replaces M O . <strong>The</strong> expression r F n is known as a triplescalar product (see item 8 in Art. C/7, Appendix C). It need not <strong>be</strong> written(r F) n <strong>be</strong>cause a cross product cannot <strong>be</strong> formed <strong>by</strong> a vector anda scalar. Thus, the association r (F n) would have no meaning.<strong>The</strong> triple scalar product may <strong>be</strong> represented <strong>by</strong> the determinantFigure 2/23r xM M F xr yF yr z F z(2/17)where , , are the direction cosines <strong>of</strong> the unit vector n.Varignon’s <strong>The</strong>orem in Three DimensionsIn Art. 2/4 we introduced Varignon’s theorem in two dimensions.<strong>The</strong> theorem is easily extended to three dimensions. Figure 2/24 shows asystem <strong>of</strong> concurrent <strong>force</strong>s F 1 , F 2 , F 3 ,.... <strong>The</strong> sum <strong>of</strong> the momentsabout O <strong>of</strong> these <strong>force</strong>s isOrF 3AF 2F 1r F 1 r F 2 r F 3 r (F 1 F 2 F 3 ) r ΣFFigure 2/24


c02.qxd 10/29/07 1:38 PM Page 7676 Chapter 2 Force Systemswhere we have used the distributive law for cross products. Using thesymbol M O to represent the sum <strong>of</strong> the moments on the left side <strong>of</strong> theabove equation, we haveM O Σ(r F) r R(2/18)This equation states that the sum <strong>of</strong> the moments <strong>of</strong> a system <strong>of</strong> concurrent<strong>force</strong>s about a given point equals the moment <strong>of</strong> their sum aboutthe same point. As mentioned in Art. 2/4, this principle has many applicationsin mechanics.Couples in Three Dimensions<strong>The</strong> concept <strong>of</strong> the couple was introduced in Art. 2/5 and is easilyextended to three dimensions. Figure 2/25 shows two equal and opposite<strong>force</strong>s F and F acting on a body. <strong>The</strong> vector r runs from any point Bon the line <strong>of</strong> action <strong>of</strong> F to any point A on the line <strong>of</strong> action <strong>of</strong> F.Points A and B are located <strong>by</strong> position vectors r A and r B from any pointO. <strong>The</strong> combined moment <strong>of</strong> the two <strong>force</strong>s about O isB–FrMAdFr Br AM r A F r B (F) (r A r B ) FOFigure 2/25However, r A r B r, so that all reference to the moment center O disappears,and the moment <strong>of</strong> the couple <strong>be</strong>comesM r F(2/19)Thus, the moment <strong>of</strong> a couple is the same about all points. <strong>The</strong> magnitude<strong>of</strong> M is M Fd, where d is the perpendicular distance <strong>be</strong>tween thelines <strong>of</strong> action <strong>of</strong> the two <strong>force</strong>s, as descri<strong>be</strong>d in Art. 2/5.<strong>The</strong> moment <strong>of</strong> a couple is a free vector, whereas the moment <strong>of</strong> a<strong>force</strong> about a point (which is also the moment about a defined axisthrough the point) is a sliding vector whose direction is along the axisthrough the point. As in the case <strong>of</strong> two dimensions, a couple tends toproduce a pure rotation <strong>of</strong> the body about an axis normal to the plane <strong>of</strong>the <strong>force</strong>s which constitute the couple.Couple vectors o<strong>be</strong>y all <strong>of</strong> the rules which govern vector quantities.Thus, in Fig. 2/26 the couple vector M 1 due to F 1 and F 1 may <strong>be</strong> addedM 1MM 1M 2F 2–F 1–FF 1≡M 2F–F 2Figure 2/26


c02.qxd 10/29/07 1:38 PM Page 77Article 2/8 Moment and Couple 77as shown to the couple vector M 2 due to F 2 and F 2 to produce the coupleM, which, in turn, can <strong>be</strong> produced <strong>by</strong> F and F.In Art. 2/5 we learned how to replace a <strong>force</strong> <strong>by</strong> its equivalent<strong>force</strong>–couple system. You should also <strong>be</strong> able to carry out this replacementin three dimensions. <strong>The</strong> procedure is represented in Fig. 2/27,where the <strong>force</strong> F acting on a rigid body at point A is replaced <strong>by</strong> anequal <strong>force</strong> at point B and the couple M r F. By adding the equaland opposite <strong>force</strong>s F and F at B, we obtain the couple composed <strong>of</strong>F and the original F. Thus, we see that the couple vector is simply themoment <strong>of</strong> the original <strong>force</strong> about the point to which the <strong>force</strong> is <strong>be</strong>ingmoved. We emphasize that r is a vector which runs from B to any pointon the line <strong>of</strong> action <strong>of</strong> the original <strong>force</strong> passing through A.M = r × FFFBA≡BFrA≡BF–FFigure 2/27


c02.qxd 10/29/07 1:38 PM Page 7878 Chapter 2 Force SystemsSample Problem 2/11zDetermine the moment <strong>of</strong> <strong>force</strong> F about point O (a) <strong>by</strong> inspection and (b) <strong>by</strong>the formal cross-product definition M O r F.abFSolution. (a) Because F is parallel to the y-axis, F has no moment about thataxis. It should <strong>be</strong> clear that the moment arm from the x-axis to the line <strong>of</strong> action<strong>of</strong> F is c and that the moment <strong>of</strong> F about the x-axis is negative. Similarly, themoment arm from the z-axis to the line <strong>of</strong> action <strong>of</strong> F is a and the moment <strong>of</strong> Fabout the z-axis is positive. So we havexcOyM O cFi aFk F(ci ak)Ans.z(b) Formally,abM O r F (ai ck) Fj aFk cFiFHelpful Hint F(ci ak)Ans.crOy Again we stress that r runs from the moment center to the line <strong>of</strong> action <strong>of</strong> F.Another permissible, but less convenient, position vector is r ai bj ck.xSample Problem 2/121.6 mz<strong>The</strong> turnbuckle is tightened until the tension in cable AB is 2.4 kN. Determinethe moment about point O <strong>of</strong> the cable <strong>force</strong> acting on point A and themagnitude <strong>of</strong> this moment.A2 mSolution.We <strong>be</strong>gin <strong>by</strong> writing the descri<strong>be</strong>d <strong>force</strong> as a vector.T Tn AB 2.4 0.8i 1.5j 2k0.8 2 1.5 2 2 2 0.731i 1.371j 1.829k kNx1.5 mBO0.8 mzy<strong>The</strong> moment <strong>of</strong> this <strong>force</strong> about point O is1.6 mM O r OA T (1.6i 2k) (0.731i 1.371j 1.829k)A 2.74i 4.39j 2.19k kN mAns.r OAThis vector has a magnitude2 mTM O 2.74 2 4.39 2 2.19 2 5.62 kN mAns.Helpful Hint <strong>The</strong> student should verify <strong>by</strong> inspection the signs <strong>of</strong> the moment components.x1.5 mBO0.8 my


c02.qxd 10/29/07 1:38 PM Page 79Article 2/8 Moment and Couple 79Sample Problem 2/13yA tension T <strong>of</strong> magnitude 10 kN is applied to the cable attached to the top A<strong>of</strong> the rigid mast and secured to the ground at B. Determine the moment M z <strong>of</strong> Tabout the z-axis passing through the base O.ASolution (a). <strong>The</strong> required moment may <strong>be</strong> obtained <strong>by</strong> finding the componentalong the z-axis <strong>of</strong> the moment M O <strong>of</strong> T about point O. <strong>The</strong> vector M O isnormal to the plane defined <strong>by</strong> T and point O, as shown in the accompanying figure.In the use <strong>of</strong> Eq. 2/14 to find M O , the vector r is any vector from point O tothe line <strong>of</strong> action <strong>of</strong> T. <strong>The</strong> simplest choice is the vector from O to A, which iswritten as r 15j m. <strong>The</strong> vector expression for T isFrom Eq. 2/14,[M O r F]T Tn AB 10 12i 15j 9k(12) 2 (15) 2 (9) 2 10(0.566i 0.707j 0.424k) kNM O 15j 10(0.566i 0.707j 0.424k) 150(0.566k 0.424i) kN m<strong>The</strong> value M z <strong>of</strong> the desired moment is the scalar component <strong>of</strong> M O in thez-direction or M z M O k. <strong>The</strong>refore,M z 150(0.566k 0.424i) k 84.9 kN mAns.<strong>The</strong> minus sign indicates that the vector M z is in the negative z-direction. Expressedas a vector, the moment is M z 84.9k kN m.z15 mO12 mHelpful HintsT = 10 kNB9 m We could also use the vector from Oto B for r and obtain the same result,but using vector OA is simpler. It is always helpful to accompany yourvector operations with a sketch <strong>of</strong> thevectors so as to retain a clear picture<strong>of</strong> the geometry <strong>of</strong> the problem. Sketch the x-y view <strong>of</strong> the problemand show d.yxSolution (b). <strong>The</strong> <strong>force</strong> <strong>of</strong> magnitude T is resolved into components T z and T xyin the x-y plane. Since T z is parallel to the z-axis, it can exert no moment about this axis. <strong>The</strong> moment M z is, then, due only to T xy and is M z T xy d, where d isthe perpendicular distance from T xy to O. <strong>The</strong> cosine <strong>of</strong> the angle <strong>be</strong>tween T andT xy is 15 2 12 2 / 15 2 12 2 9 2 0.906, and therefore,T xy 10(0.906) 9.06 kN<strong>The</strong> moment arm d equals OA multiplied <strong>by</strong> the sine <strong>of</strong> the angle <strong>be</strong>tween T xyand OA, orzArOTM zBxM od 151212 2 15 2 9.37 mHence, the moment <strong>of</strong> T about the z-axis has the magnitudeyAT xM z 9.06(9.37) 84.9 kN mAns.and is clockwise when viewed in the x-y plane.T15 mSolution (c). <strong>The</strong> component T xy is further resolved into its components T x and T y .It is clear that T y exerts no moment about the z-axis since it passes through it, sothat the required moment is due to T x alone. <strong>The</strong> direction cosine <strong>of</strong> T with respectto the x-axis is 12/9 2 12 2 15 2 0.566 so that T x 10(0.566) 5.66 kN. Thus,M z 5.66(15) 84.9 kN mAns.zT zT yT xyxO12 mB9 m


c02.qxd 10/29/07 1:38 PM Page 8080 Chapter 2 Force SystemsSample Problem 2/14Determine the magnitude and direction <strong>of</strong> the couple M which will replacethe two given couples and still produce the same external effect on the block.Specify the two <strong>force</strong>s F and F, applied in the two faces <strong>of</strong> the block parallel tothe y-z plane, which may replace the four given <strong>force</strong>s. <strong>The</strong> 30-N <strong>force</strong>s act parallelto the y-z plane.x 100 mm50 mm30 N60°60 mm30 N60°40 mmy25 NSolution. <strong>The</strong> couple due to the 30-N <strong>force</strong>s has the magnitude M 1 30(0.06) 1.80 N m. <strong>The</strong> direction <strong>of</strong> M 1 is normal to the plane defined <strong>by</strong> the two <strong>force</strong>s,and the sense, shown in the figure, is established <strong>by</strong> the right-hand convention.<strong>The</strong> couple due to the 25-N <strong>force</strong>s has the magnitude M 2 25(0.10) 2.50 N mwith the direction and sense shown in the same figure. <strong>The</strong> two couple vectorscombine to give the components25 NM 2 = 2.5 N·mzThus,withAns.Ans.<strong>The</strong> <strong>force</strong>s F and F lie in a plane normal to the couple M, and their momentarm as seen from the right-hand figure is 100 mm. Thus, each <strong>force</strong> has themagnitude[M = Fd] F 2.230.10 22.3 NAns.and the direction 44.3.M y 1.80 sin 60 1.559 N mM z 2.50 1.80 cos 60 1.600 N mM (1.559) 2 (1.600) 2 2.23 N m tan1 1.5591.600 tan1 0.974 44.3zθ60°MyxM 1 = 1.8 N·mHelpful Hintθ–F Bear in mind that the couple vectorsare free vectors and therefore haveno unique lines <strong>of</strong> action.zθFySample Problem 2/15zA <strong>force</strong> <strong>of</strong> 400 N is applied at A to the handle <strong>of</strong> the control lever which is attachedto the fixed shaft OB. In determining the effect <strong>of</strong> the <strong>force</strong> on the shaftat a cross section such as that at O, we may replace the <strong>force</strong> <strong>by</strong> an equivalent<strong>force</strong> at O and a couple. Descri<strong>be</strong> this couple as a vector M.Solution. <strong>The</strong> couple may <strong>be</strong> expressed in vector notation as M r F,where r OA l 0.2j 0.125k m and F 400i N. Thus,M (0.2j 0.125k) (400i) 50j 80k N m75 mmOxBz200 mmyA400 N50 mmAlternatively we see that moving the 400-N <strong>force</strong> through a distance d 0.125 2 0.2 2 0.236 m to a parallel position through O requires the addition<strong>of</strong> a couple M whose magnitude isM Fd 400(0.236) 94.3 N mAns.<strong>The</strong> couple vector is perpendicular to the plane in which the <strong>force</strong> is shifted, andits sense is that <strong>of</strong> the moment <strong>of</strong> the given <strong>force</strong> about O. <strong>The</strong> direction <strong>of</strong> M inthe y-z plane is given <strong>by</strong>xMθO400 Ndθ200 mm(400 N)A125 mmy1 125 tan200 32.0Ans.


c02.qxd 10/29/07 1:38 PM Page 81Article 2/8 Problems 81PROBLEMSIntroductory Problems2/117 Determine the moment <strong>of</strong> <strong>force</strong> F about point O.Ans. M O F(cj bk)z2/119 Determine the moment about O <strong>of</strong> the <strong>force</strong> <strong>of</strong>magnitude F for the case (a) when the <strong>force</strong> F isapplied at A and for the case (b) when F is appliedat B.Ans. (a) M O FLi(b) M O F(Li Dk)ycaObFF(b)BF(a)ALOxyDzxProblem 2/1172/118 Determine the moment <strong>of</strong> <strong>force</strong> F about point A.xProblem 2/1192/120 In opening a door which is equipped with a heavydutyreturn mechanism, a person exerts a <strong>force</strong> P<strong>of</strong> magnitude 32 N as shown. Force P and the normaln to the face <strong>of</strong> the door lie in a vertical plane.Compute the moment <strong>of</strong> P about the z-axis.zAaFbz900 mmPy30°Problem 2/118ny20°xProblem 2/120


c02.qxd 10/29/07 1:38 PM Page 8282 Chapter 2 Force Systems2/121 <strong>The</strong> two <strong>force</strong>s acting on the handles <strong>of</strong> the pipewrenches constitute a couple M. Express the coupleas a vector.Ans. M 75i 22.5j N m250 mm150 mm250 mm150 Ny2/123 A helicopter is shown here with certain threedimensionalgeometry given. During a ground test,a 400-N aerodynamic <strong>force</strong> is applied to the tailrotor at P as shown. Determine the moment <strong>of</strong> this<strong>force</strong> about point O <strong>of</strong> the airframe.Ans. M O 480i 2400k N mz6 mx0.8 mxO150 NProblem 2/121y1.2 m400 NP2/122 <strong>The</strong> <strong>be</strong>nt bar has a mass per unit <strong>of</strong> length. Determinethe moment <strong>of</strong> the weight <strong>of</strong> the bar aboutpoint O.zProblem 2/1232/124 <strong>The</strong> 24-N <strong>force</strong> is applied at point A <strong>of</strong> the crank assembly.Determine the moment <strong>of</strong> this <strong>force</strong> aboutpoint O.dzF = 24 NA30°hO18 mmyOyx36 mm65°Problem 2/124xProblem 2/122


c02.qxd 10/29/07 1:38 PM Page 83Article 2/8 Problems 832/125 <strong>The</strong> right-angle pipe OAB <strong>of</strong> Prob. 2/108 is shownagain here. Replace the 750-N tensile <strong>force</strong> whichthe cable exerts on point B <strong>by</strong> a <strong>force</strong>–couple systemat point O.Ans. R 598i 411j 189.5k NM O 361i 718j 419k N m2/127 If the magnitude <strong>of</strong> the moment <strong>of</strong> F about line CDis 50 N m, determine the magnitude <strong>of</strong> F.Ans. F 228 NByFzC0.7 m1.2 mO1.6 m30°A0.8 mx0.2 mC0.2 mA0.2 m 0.4 mProblem 2/127Representative ProblemsDProblem 2/1252/126 Determine the moment associated with the pair <strong>of</strong>400-N <strong>force</strong>s applied to the T-shaped structure.B2/128 A mechanic applies the horizontal 100-N <strong>force</strong> perpendicularto the wrench as indicated in the figure.Determine the moment <strong>of</strong> this <strong>force</strong> about the boltcenter O. <strong>The</strong> wrench centerline lies in the x-y plane.yyOx15°B400 NzO100 NA15°x0.45 mA0.25 m0.25 mzOA = 185 mmProblem 2/128400 N15°Problem 2/126


c02.qxd 10/29/07 1:38 PM Page 8484 Chapter 2 Force Systems2/129 Two 4-N thrusters on the nonrotating satellite aresimultaneously fired as shown. Compute the momentassociated with this couple and state aboutwhich satellite axes rotations will <strong>be</strong>gin to occur.Ans. M 5i 4k N m625 mmx4 N500mmzG500mm4 N625 mmy2/131 A space shuttle orbiter is subjected to thrusts fromfive <strong>of</strong> the engines <strong>of</strong> its reaction control system.Four <strong>of</strong> the thrusts are shown in the figure; thefifth is an 850-N upward thrust at the right rear,symmetric to the 850-N thrust shown on the leftrear. Compute the moment <strong>of</strong> these <strong>force</strong>s aboutpoint G and show that the <strong>force</strong>s have the samemoment about all points.Ans. M 3400i 51 000j 51 000k N m1700 N1700 NyG3.2 m3.2 m1700 N850 N12 m2 mProblem 2/1292/130 Compute the moment M O <strong>of</strong> the 1.2-kN <strong>force</strong> aboutthe axis O-O.O300 mm60°40°1.2 kNxProblem 2/1312/132 In picking up a load from position B, a cable tensionT <strong>of</strong> magnitude 24 kN is developed. Calculatethe moment which T produces about the base O <strong>of</strong>the construction crane.zz18 m18 m105 mm200 mm OAProblem 2/130T30 mxOBy5 m 6 mProblem 2/132


c02.qxd 10/29/07 1:38 PM Page 85Article 2/8 Problems 852/133 <strong>The</strong> specialty wrench shown is used for difficult-toaccessbolts which have a low torque specification.Determine the moment about O <strong>of</strong> the 40-N <strong>force</strong>applied at point A <strong>of</strong> the wrench handle. Discussany shortcomings <strong>of</strong> this wrench.Ans. M O 12i 9k N mAzy125 mmO100mm125mmz40 NAx300 mmx100 mm50 mmB50 mm60°400 NyProblem 2/1332/134 In order to decrease the undesirable moment aboutthe x-axis, a mechanic uses his left hand to supportthe wrench handle at point B. What upward <strong>force</strong> F<strong>must</strong> he exert in order that there <strong>be</strong> no momentabout the x-axis? What then is the net momentabout O?yProblem 2/1352/136 <strong>The</strong> specialty wrench shown in the figure is designedfor access to the hold-down bolt on certainautomobile distributors. For the configurationshown where the wrench lies in a vertical planeand a horizontal 200-N <strong>force</strong> is applied at A perpendicularto the handle, calculate the moment M O appliedto the bolt at O. For what value <strong>of</strong> thedistance d would the z-component <strong>of</strong> M O <strong>be</strong> zero?OA100mm125mmzB40 N FAx150 mm300 mm200 Ν20°d = 125 mmProblem 2/134200 mm2/135 Determine the moment <strong>of</strong> the 400-N <strong>force</strong> aboutpoint A <strong>by</strong> (a) using the vector cross-product relationand (b) resolving the <strong>force</strong> into its componentsand finding their respective moments.Ans. M A 9.64i 17.32j 10k N mxzO70 mmyProblem 2/136


c02.qxd 10/29/07 1:38 PM Page 8686 Chapter 2 Force Systems2/137 <strong>The</strong> moment M applied to the pulley wheel causesa tension T 80 N in the cable which is secured tothe wheel at D and to the ground at E. Determinethe moment about O <strong>of</strong> this 80-N <strong>force</strong> as appliedat C.Ans. M O 15.49i 4k N mAz200 mmDM2/139 Using the principles to <strong>be</strong> developed in Chapter 3on equilibrium, one can determine that the tensionin cable AB is 143.4 N. Determine the momentabout the x-axis <strong>of</strong> this tension <strong>force</strong> acting onpoint A. Compare your result with the moment <strong>of</strong>the weight W <strong>of</strong> the 15-kg uniform plate about thex-axis. What is the moment <strong>of</strong> the tension <strong>force</strong> actingat A about line OB?Ans. M x 31.1 N m, (M x ) W 31.1 N mM OB 0zCB100 mmB0.35 mO400 mm0.4 m20°yOG15 kgxEyx0.45 m0.7 mProblem 2/137AW2/138 An 320-N <strong>force</strong> is applied to the end <strong>of</strong> the wrenchas the sprocket wheel is attached to an enginecrankshaft. <strong>The</strong> <strong>force</strong> is perpendicular to the planecontaining points O, A, and B. Determine the moment<strong>of</strong> this <strong>force</strong> about point O.Problem 2/139F = 320 Ny20°x10°B400 mmO100mmAzProblem 2/138


c02.qxd 10/29/07 1:38 PM Page 87Article 2/8 Problems 872/140 <strong>The</strong> rigid pole and cross-arm assembly <strong>of</strong> Prob.2/105 is shown again here. Determine the vectorexpression for the moment <strong>of</strong> the 1.2-kN tension(a) about point O and (b) about the pole z-axis. Findeach moment in two different ways.Fxx′1 m2 mGB1 mAzO1.5 m1.5 m3 mz′1.5 m3 mCEy′T = 1.2 kNDy2/142 Determine the vector expression for the momentM O <strong>of</strong> the 600-N <strong>force</strong> about point O. <strong>The</strong> designspecification for the bolt at O would require thisresult.50 mmz150 mmy60° A130 mm600 NO45°Problem 2/142140 mmxProblem 2/1402/141 A 5-N vertical <strong>force</strong> is applied to the knob <strong>of</strong> thewindow-opener mechanism when the crank BC ishorizontal. Determine the moment <strong>of</strong> the <strong>force</strong>about point A and about line AB.Ans. M A 375i 325j N mmM AB 281i 162.4k N mm25mmz50 mm75 mmyC5 NDAB30°xProblem 2/141


c02.qxd 10/29/07 1:38 PM Page 8888 Chapter 2 Force Systems2/9 RESULTANTS© Raymond For<strong>be</strong>s/Age Fotostock America, Inc.In Art. 2/6 we defined the resultant as the simplest <strong>force</strong> combinationwhich can replace a given system <strong>of</strong> <strong>force</strong>s without altering the externaleffect on the rigid body on which the <strong>force</strong>s act. We found themagnitude and direction <strong>of</strong> the resultant <strong>force</strong> for the two-dimensional<strong>force</strong> system <strong>by</strong> a vector summation <strong>of</strong> <strong>force</strong>s, Eq. 2/9, and we located theline <strong>of</strong> action <strong>of</strong> the resultant <strong>force</strong> <strong>by</strong> applying the principle <strong>of</strong> moments,Eq. 2/10. <strong>The</strong>se same principles can <strong>be</strong> extended to three dimensions.In the previous article we showed that a <strong>force</strong> could <strong>be</strong> moved to aparallel position <strong>by</strong> adding a corresponding couple. Thus, for the system<strong>of</strong> <strong>force</strong>s F 1 , F 2 , F 3 . . . acting on a rigid body in Fig. 2/28a, we may moveeach <strong>of</strong> them in turn to the arbitrary point O, provided we also introducea couple for each <strong>force</strong> transferred. Thus, for example, we maymove <strong>force</strong> F 1 to O, provided we introduce the couple M 1 r 1 F 1 ,where r 1 is a vector from O to any point on the line <strong>of</strong> action <strong>of</strong> F 1 .When all <strong>force</strong>s are shifted to O in this manner, we have a system <strong>of</strong>concurrent <strong>force</strong>s at O and a system <strong>of</strong> couple vectors, as represented inpart b <strong>of</strong> the figure. <strong>The</strong> concurrent <strong>force</strong>s may then <strong>be</strong> added vectoriallyto produce a resultant <strong>force</strong> R, and the couples may also <strong>be</strong> added toproduce a resultant couple M, Fig. 2/28c. <strong>The</strong> general <strong>force</strong> system,then, is reduced to<strong>The</strong> cables <strong>of</strong> a suspension bridgeexert a three-dimensional system <strong>of</strong>concentrated <strong>force</strong>s on this bridgetower.R F 1 F 2 F 3 ΣFM M 1 M 2 M 3 Σ(r F)(2/20)<strong>The</strong> couple vectors are shown through point O, but <strong>be</strong>cause they arefree vectors, they may <strong>be</strong> represented in any parallel positions. <strong>The</strong>magnitudes <strong>of</strong> the resultants and their components areR x ΣF x R y ΣF y R z ΣF zR (ΣF x ) 2 (ΣF y ) 2 (ΣF z ) 2M x Σ(r F) x M y Σ(r F) y M z Σ(r F) zM M x2 M y2 M z2(2/21)M2M 3F 1F 2F 1M 1F 2MRO≡O≡OF 3F 3(a)(b)Figure 2/28(c)


c02.qxd 10/29/07 1:38 PM Page 89Article 2/9 Resultants 89<strong>The</strong> point O selected as the point <strong>of</strong> concurrency for the <strong>force</strong>s is arbitrary,and the magnitude and direction <strong>of</strong> M depend on the particularpoint O selected. <strong>The</strong> magnitude and direction <strong>of</strong> R, however, are thesame no matter which point is selected.In general, any system <strong>of</strong> <strong>force</strong>s may <strong>be</strong> replaced <strong>by</strong> its resultant<strong>force</strong> R and the resultant couple M. In dynamics we usually select themass center as the reference point. <strong>The</strong> change in the linear motion <strong>of</strong>the body is determined <strong>by</strong> the resultant <strong>force</strong>, and the change in the angularmotion <strong>of</strong> the body is determined <strong>by</strong> the resultant couple. In statics,the body is in complete equilibrium when the resultant <strong>force</strong> R iszero and the resultant couple M is also zero. Thus, the determination <strong>of</strong>resultants is essential in both statics and dynamics.We now examine the resultants for several special <strong>force</strong> <strong>systems</strong>.Concurrent Forces. When <strong>force</strong>s are concurrent at a point, only thefirst <strong>of</strong> Eqs. 2/20 needs to <strong>be</strong> used <strong>be</strong>cause there are no moments aboutthe point <strong>of</strong> concurrency.Parallel Forces. For a system <strong>of</strong> parallel <strong>force</strong>s not all in the sameplane, the magnitude <strong>of</strong> the parallel resultant <strong>force</strong> R is simply the magnitude<strong>of</strong> the algebraic sum <strong>of</strong> the given <strong>force</strong>s. <strong>The</strong> position <strong>of</strong> its line <strong>of</strong>action is obtained from the principle <strong>of</strong> moments <strong>by</strong> requiring thatr R M O . Here r is a position vector extending from the <strong>force</strong>–couplereference point O to the final line <strong>of</strong> action <strong>of</strong> R, and M O is the sum <strong>of</strong>the moments <strong>of</strong> the individual <strong>force</strong>s about O. See Sample Problem 2/17for an example <strong>of</strong> parallel-<strong>force</strong> <strong>systems</strong>.Coplanar Forces. Article 2/6 was devoted to this <strong>force</strong> system.Wrench Resultant. When the resultant couple vector M is parallelto the resultant <strong>force</strong> R, as shown in Fig. 2/29, the resultant is called awrench. By definition a wrench is positive if the couple and <strong>force</strong> vectorspoint in the same direction and negative if they point in opposite directions.A common example <strong>of</strong> a positive wrench is found with the application<strong>of</strong> a screwdriver, to drive a right-handed screw. Any general <strong>force</strong>system may <strong>be</strong> represented <strong>by</strong> a wrench applied along a unique line <strong>of</strong>action. This reduction is illustrated in Fig. 2/30, where part a <strong>of</strong> the figurerepresents, for the general <strong>force</strong> system, the resultant <strong>force</strong> R actingat some point O and the corresponding resultant couple M. Although Mis a free vector, for convenience we represent it as acting through O.In part b <strong>of</strong> the figure, M is resolved into components M 1 along the direction<strong>of</strong> R and M 2 normal to R. In part c <strong>of</strong> the figure, the couple M 2 isreplaced <strong>by</strong> its equivalent <strong>of</strong> two <strong>force</strong>s R and R separated <strong>by</strong> a distanceMRMRPositive wrenchNegative wrenchFigure 2/29


c02.qxd 10/29/07 1:38 PM Page 9090 Chapter 2 Force SystemsMMM 2M 1 RROO(a)(b)OdM 1RROR–RM 1(c)(d)Figure 2/30d M 2 /R with R applied at O to cancel the original R. This step leavesthe resultant R, which acts along a new and unique line <strong>of</strong> action, and theparallel couple M 1 , which is a free vector, as shown in part d <strong>of</strong> the figure.Thus, the resultants <strong>of</strong> the original general <strong>force</strong> system have <strong>be</strong>en transformedinto a wrench (positive in this illustration) with its unique axis defined<strong>by</strong> the new position <strong>of</strong> R.We see from Fig. 2/30 that the axis <strong>of</strong> the wrench resultant lies in aplane through O normal to the plane defined <strong>by</strong> R and M. <strong>The</strong> wrench isthe simplest form in which the resultant <strong>of</strong> a general <strong>force</strong> system may<strong>be</strong> expressed. This form <strong>of</strong> the resultant, however, has limited application,<strong>be</strong>cause it is usually more convenient to use as the reference pointsome point O such as the mass center <strong>of</strong> the body or another convenientorigin <strong>of</strong> coordinates not on the wrench axis.


c02.qxd 10/29/07 1:38 PM Page 91Article 2/9 Resultants 91Sample Problem 2/16zDetermine the resultant <strong>of</strong> the <strong>force</strong> and couple system which acts on therectangular solid.50 N70 N . m100 N . mSolution. We choose point O as a convenient reference point for the initialstep <strong>of</strong> reducing the given <strong>force</strong>s to a <strong>force</strong>–couple system. <strong>The</strong> resultant <strong>force</strong> isR ΣF (80 80)i (100 100)j (50 50)k 0 N<strong>The</strong> sum <strong>of</strong> the moments about O is80 N80 NO50 N96N . m1.2 mM O [50(1.6) 70]i [80(1.2) 96]j [100(1) 100]k 10i N mx1.6 m1 m 100 NHence, the resultant consists <strong>of</strong> a couple, which <strong>of</strong> course may <strong>be</strong> applied at anypoint on the body or the body extended.100 NyHelpful Hints Since the <strong>force</strong> summation is zero, we conclude that the resultant, if it exists,<strong>must</strong> <strong>be</strong> a couple. <strong>The</strong> moments associated with the <strong>force</strong> pairs are easily obtained <strong>by</strong> using theM Fd rule and assigning the unit-vector direction <strong>by</strong> inspection. In manythree-dimensional problems, this may <strong>be</strong> simpler than the M rFapproach.Sample Problem 2/1750 NDetermine the resultant <strong>of</strong> the system <strong>of</strong> parallel <strong>force</strong>s which act on theplate. Solve with a vector approach.x0.5 mz0.5 mSolution.Transfer <strong>of</strong> all <strong>force</strong>s to point O results in the <strong>force</strong>–couple systemO0.35 mR ΣF (200 500 300 50)j 350j N500 N0.35 mM O [50(0.35) 300(0.35)]i [50(0.50) 200(0.50)]k 87.5i 125k N m<strong>The</strong> placement <strong>of</strong> R so that it alone represents the above <strong>force</strong>–couple system isdetermined <strong>by</strong> the principle <strong>of</strong> moments in vector formr R M O(xi yj zk) 350j 87.5i 125k350xk 350zi 87.5i 125kFrom the one vector equation we may obtain the two scalar equations350x 125 and 350z 87.5y300 NxRRy200 Nzxzr OM OHence, x 0.357 m and z 0.250 m are the coordinates through which theline <strong>of</strong> action <strong>of</strong> R <strong>must</strong> pass. <strong>The</strong> value <strong>of</strong> y may, <strong>of</strong> course, <strong>be</strong> any value, aspermitted <strong>by</strong> the principle <strong>of</strong> transmissibility. Thus, as expected, the variable ydrops out <strong>of</strong> the above vector analysis.Helpful Hint You should also carry out a scalarsolution to this problem.


c02.qxd 10/29/07 1:38 PM Page 9292 Chapter 2 Force SystemsSample Problem 2/18Replace the two <strong>force</strong>s and the negative wrench <strong>by</strong> a single <strong>force</strong> R appliedat A and the corresponding couple M.Solution.<strong>The</strong> resultant <strong>force</strong> has the components[R x ΣF x ] R x 500 sin 40 700 sin 60 928 N[R y ΣF y ] R y 600 500 cos 40 cos 45 871 N[R z ΣF z ] R z 700 cos 60 500 cos 40 sin 45 621 N100mmx30mmA50 mm60 mm120 mm700 N60°500 N25 N·m40°z B80 mm45°40 mm 600 NyThus,andR 928i 871j 621k NR (928) 2 (871) 2 (621) 2 1416 NAns.MR<strong>The</strong> couple to <strong>be</strong> added as a result <strong>of</strong> moving the 500-N <strong>force</strong> is[M r F] M 500 (0.08i 0.12j 0.05k) 500(i sin 40 j cos 40 cos 45 k cos 40 sin 45)where r is the vector from A to B.A<strong>The</strong> term-<strong>by</strong>-term, or determinant, expansion givesM 500 18.95i 5.59j 16.90k N m<strong>The</strong> moment <strong>of</strong> the 600-N <strong>force</strong> about A is written <strong>by</strong> inspection <strong>of</strong> its x- and z-components, which givesM 600 (600)(0.060)i (600)(0.040)k 36.0i 24.0k N m<strong>The</strong> moment <strong>of</strong> the 700-N <strong>force</strong> about A is easily obtained from the moments <strong>of</strong>the x- and z-components <strong>of</strong> the <strong>force</strong>. <strong>The</strong> result <strong>be</strong>comesM 700 (700 cos 60)(0.030)i [(700 sin 60)(0.060) (700 cos 60)(0.100)]j (700 sin 60)(0.030)k 10.5i 71.4j 18.19k N mAlso, the couple <strong>of</strong> the given wrench may <strong>be</strong> writtenM 25.0(i sin 40 j cos 40 cos 45 k cos 40 sin 45) 16.07i 13.54j 13.54k N m<strong>The</strong>refore, the resultant couple on adding together the i-, j-, and k-terms <strong>of</strong> thefour M’s isM 49.4i 90.5j 24.6k N mHelpful Hints Suggestion: Check the cross-productresults <strong>by</strong> evaluating the momentsabout A <strong>of</strong> the components <strong>of</strong> the500-N <strong>force</strong> directly from the sketch. For the 600-N and 700-N <strong>force</strong>s it iseasier to obtain the components <strong>of</strong>their moments about the coordinatedirections through A <strong>by</strong> inspection<strong>of</strong> the figure than it is to set up thecross-product relations. <strong>The</strong> 25-N m couple vector <strong>of</strong> thewrench points in the direction oppositeto that <strong>of</strong> the 500-N <strong>force</strong>, andwe <strong>must</strong> resolve it into its x-, y-, andz-components to <strong>be</strong> added to theother couple-vector components. Although the resultant couple vectorM in the sketch <strong>of</strong> the resultantsis shown through A, we recognizethat a couple vector is a free vectorand therefore has no specified line<strong>of</strong> action.andM (49.4) 2 (90.5) 2 (24.6) 2 106.0 N mAns.


c02.qxd 10/29/07 1:38 PM Page 93Article 2/9 Resultants 93Sample Problem 2/19Determine the wrench resultant <strong>of</strong> the three <strong>force</strong>s acting on the bracket.Calculate the coordinates <strong>of</strong> the point P in the x-y plane through which the resultant<strong>force</strong> <strong>of</strong> the wrench acts. Also find the magnitude <strong>of</strong> the couple M <strong>of</strong> thewrench.Solution. <strong>The</strong> direction cosines <strong>of</strong> the couple M <strong>of</strong> the wrench <strong>must</strong> <strong>be</strong> the same as those <strong>of</strong> the resultant <strong>force</strong> R, assuming that the wrench is positive. <strong>The</strong>resultant <strong>force</strong> is40 N20 Nz80 mm60 mmy100 mmxPy40 NxR 20i 40j 40k Nand its direction cosines areR (20) 2 (40) 2 (40) 2 60 Nz80 mmcos x 20/60 1/3 cos y 40/60 2/3 cos z 40/60 2/360 mmyR<strong>The</strong> moment <strong>of</strong> the wrench couple <strong>must</strong> equal the sum <strong>of</strong> the moments <strong>of</strong>the given <strong>force</strong>s about point P through which R passes. <strong>The</strong> moments about P <strong>of</strong>the three <strong>force</strong>s are(M) Rx 20yk N mm(M) Ry 40(60)i 40xk N mmx = 60 mmP100 mmMxy = 40 mm(M) Rz 40(80 y)i 40(100 x)j N mmand the total moment isM (800 40y)i (4000 40x)j (40x 20y)k N mm<strong>The</strong> direction cosines <strong>of</strong> M areHelpful Hint We assume initially that the wrenchis positive. If M turns out to <strong>be</strong> negative,then the direction <strong>of</strong> the couplevector is opposite to that <strong>of</strong> theresultant <strong>force</strong>.cos x (800 40y)/Mcos y (4000 40x)/Mcos z (40x 20y)/Mwhere M is the magnitude <strong>of</strong> M. Equating the direction cosines <strong>of</strong> R and M gives800 40y M 34000 40x 2M 340x 20y 2M 3Solution <strong>of</strong> the three equations givesM 2400 N mm x 60 mm y 40 mmAns.We see that M turned out to <strong>be</strong> negative, which means that the couple vector ispointing in the direction opposite to R, which makes the wrench negative.


c02.qxd 10/29/07 1:38 PM Page 9494 Chapter 2 Force SystemsPROBLEMSIntroductory Problems2/143 Three <strong>force</strong>s act at point O. If it is known that they-component <strong>of</strong> the resultant R is 5 kN and thatthe z-component is 6 kN, determine F 3 , , and R.Ans. F 3 10.82 kN, 33.7, R 10.49 kN2/145 <strong>The</strong> thin rectangular plate is subjected to the four<strong>force</strong>s shown. Determine the equivalent <strong>force</strong>–couple system at O. What is the resultant <strong>of</strong> thesystem?Ans.R 0, M O Fb 1 32 izF 3zFθOF 2 = 7 kNF 1 = 4 kN—2bF—2b Ob FzxyProblem 2/1432/144 Three equal <strong>force</strong>s are exerted on the equilateralplate as shown. Reduce the <strong>force</strong> system to anequivalent <strong>force</strong>–couple system at point O. Showthat R is perpendicular to M O .Fx30°F2bF30°Problem 2/1452/146 <strong>The</strong> spacecraft <strong>of</strong> Prob. 2/129 is repeated here. <strong>The</strong>plan is to fire four 4-N thrusters as shown in orderto spin up the spacecraft about its z-axis, but thethruster at A fails. Determine the equivalent<strong>force</strong>–couple system at G for the remaining threethrusters.zObFyx<strong>by</strong>4 N4 NProblem 2/144625 mm500mmG500mm625 mmyxA4 NProblem 2/146


c02.qxd 10/29/07 1:38 PM Page 95Article 2/9 Problems 952/147 An oil tanker moves away from its docked positionunder the action <strong>of</strong> reverse thrust from screw A,forward thrust from screw B, and side thrust fromthe bow thruster C. Determine the equivalent<strong>force</strong>–couple system at the mass center G.Ans. R 8i kN, M G 48j 820k kN mz2/149 Determine the <strong>force</strong>–couple system at O which isequivalent to the two <strong>force</strong>s applied to the shaftAOB. Is R perpendicular to M O ?Ans. R 266j 1085k NM O 48.9j 114.5k N my40 my600 NA6 mCG8 kN7 m50 kN xA B50 kN5 m 5 mProblem 2/147z30°80 mmO160 mm45°Bx2/148 <strong>The</strong> pulley and gear are subjected to the loadsshown. For these <strong>force</strong>s, determine the equivalent<strong>force</strong>–couple system at point O.800 NzyO220mm75mmx330mm10°1200 N100mm800 N200 NRepresentative ProblemsProblem 2/1492/150 <strong>The</strong> commercial airliner <strong>of</strong> Prob. 2/86 is redrawnhere with three-dimensional information supplied.If engine 3 suddenly fails, determine the resultant<strong>of</strong> the three remaining engine thrust vectors, each<strong>of</strong> which has a magnitude <strong>of</strong> 90 kN. Specify the y-and z-coordinates <strong>of</strong> the point through which theline <strong>of</strong> action <strong>of</strong> the resultant passes. This informationwould <strong>be</strong> critical to the design criteria <strong>of</strong> performancewith engine failure.Problem 2/148y49 mx2 m 33 m90 kN3 m22 m12 mz90 kN12 m90 kN9 m1Problem 2/150


c02.qxd 10/29/07 1:38 PM Page 9696 Chapter 2 Force Systems2/151 Represent the resultant <strong>of</strong> the <strong>force</strong> system actingon the pipe assembly <strong>by</strong> a single <strong>force</strong> R at A and acouple M.Ans. R 120i 180j 100k NM A 100j 50k N m300 mmz120 N2/153 A 25-kg computer and a 10-kg laser printer rest ona horizontal tabletop. Determine the resultant <strong>of</strong>the system <strong>of</strong> two corresponding weights and specifythe coordinates <strong>of</strong> the point P in the x-y planethrough which the resultant passes. <strong>The</strong> twoweights act at the points G 1 and G 2 .Ans. R 343k N, M O 343i 176.6j N m(x, y) (514, 1000) mmz100 N50 N·m250 mm300 mm O10 kg25 kgx200 mm160 N100mmA160 N180 Nyx500mm1200mmBG 2600 mm yG 1A25°150mmProblem 2/1512/152 Two upward loads are exerted on the small threedimensionaltruss. Reduce these two loads to a single<strong>force</strong>–couple system at point O. Show that R isperpendicular to M O . <strong>The</strong>n determine the point inthe x-z plane through which the resultant passes.y0.9 mProblem 2/1532/154 <strong>The</strong> motor mounted on the bracket is acted on <strong>by</strong>its 160-N weight, and its shaft resists the 120-Nthrust and 25-N m couple applied to it. Determinethe resultant <strong>of</strong> the <strong>force</strong> system shown in terms <strong>of</strong>a <strong>force</strong> R at A and a couple M.z200 mm0.9 m160 N75 mmzO1.2 mAy1.2 mProblem 2/1520.9 m1600 N800 Nxx75mm25 N·m120 N100mmProblem 2/154


c02.qxd 10/29/07 1:38 PM Page 97Article 2/9 Problems 972/155 In tightening a bolt whose center is at point O, aperson exerts a 180-N <strong>force</strong> on the ratchet handlewith his right hand. In addition, with his left handhe exerts a 90-N <strong>force</strong> as shown in order to securethe socket onto the bolt head. Determine the equivalent<strong>force</strong>–couple system at O. <strong>The</strong>n find the pointin the x-y plane through which the line <strong>of</strong> action <strong>of</strong>the resultant <strong>force</strong> <strong>of</strong> the wrench passes.Ans. R 90j 180k N, M O 6.3i 36j N mx 160 mm, y 35 mmz200 mm35 mm180 N2/157 Replace the two <strong>force</strong>s acting on the frame <strong>by</strong> awrench. Write the moment associated with thewrench as a vector and specify the coordinates <strong>of</strong>the point P in the y-z plane through which the line<strong>of</strong> action <strong>of</strong> the wrench passes. Note that the <strong>force</strong><strong>of</strong> magnitude F is parallel to the x-axis.Ans. R F(i 3k), M 3aF (i 3k)10y a , z 2a10z3FaFxO90 NyProblem 2/1552a2/156 Replace the two <strong>force</strong>s acting on the pole <strong>by</strong> awrench. Write the moment M associated with thewrench as a vector and specify the coordinates <strong>of</strong>the point P in the y-z plane through which the line<strong>of</strong> action <strong>of</strong> the wrench passes.zxOyTProblem 2/157Ta3a2/158 For the system <strong>of</strong> two <strong>force</strong>s in Prob. 2/149, determinethe coordinates <strong>of</strong> the point in the x-z planethrough which the line <strong>of</strong> action <strong>of</strong> the resultant <strong>of</strong>the system passes.OyxProblem 2/156


c02.qxd 10/29/07 1:38 PM Page 9898 Chapter 2 Force Systems2/159 <strong>The</strong> resultant <strong>of</strong> the two <strong>force</strong>s and couple may <strong>be</strong>represented <strong>by</strong> a wrench. Determine the vector expressionfor the moment M <strong>of</strong> the wrench and findthe coordinates <strong>of</strong> the point P in the x-z planethrough which the resultant <strong>force</strong> <strong>of</strong> the wrenchpasses.Ans. M 10i 10j N mx z 0.1 m100 Ny300 mmz20 N·m400 mm400mmx100 N2/160 For the position shown, the crankshaft <strong>of</strong> a smalltwo-cylinder compressor is subjected to the 400-Nand 800-N <strong>force</strong>s exerted <strong>by</strong> the connecting rodsand the 200-N m couple. Replace this loading system<strong>by</strong> a <strong>force</strong>–couple system at point A. Show thatR is not perpendicular to M A . <strong>The</strong>n replace this<strong>force</strong>–couple system <strong>by</strong> a wrench. Determine themagnitude M <strong>of</strong> the moment <strong>of</strong> the wrench, themagnitude <strong>of</strong> the <strong>force</strong> R <strong>of</strong> the wrench, and the coordinates<strong>of</strong> the point in the x-z plane throughwhich the line <strong>of</strong> action <strong>of</strong> the wrench passes.Ans. M 85.8 N m, R 1108 Nx 0.1158 m, z 0.478 my24°400 N200mm24°800 NBProblem 2/159z200 N·mA200mm200mm400 mm200mmxProblem 2/160


c02.qxd 10/29/07 1:38 PM Page 99Article 2/10 Chapter Review 992/10 CHAPTER R EVIEWIn Chapter 2 we have established the <strong>properties</strong> <strong>of</strong> <strong>force</strong>s, moments,and couples, and the correct procedures for representing their effects.Mastery <strong>of</strong> this material is essential for our study <strong>of</strong> equilibrium in thechapters which follow. Failure to correctly use the procedures <strong>of</strong> Chapter2 is a common cause <strong>of</strong> errors in applying the principles <strong>of</strong> equilibrium.When difficulties arise, you should refer to this chapter to <strong>be</strong> surethat the <strong>force</strong>s, moments, and couples are correctly represented.Forces<strong>The</strong>re is frequent need to represent <strong>force</strong>s as vectors, to resolve asingle <strong>force</strong> into components along desired directions, and to combinetwo or more concurrent <strong>force</strong>s into an equivalent resultant <strong>force</strong>. Specifically,you should <strong>be</strong> able to:1. Resolve a given <strong>force</strong> vector into its components along given directions,and express the vector in terms <strong>of</strong> the unit vectors along agiven set <strong>of</strong> axes.2. Express a <strong>force</strong> as a vector when given its magnitude and informationabout its line <strong>of</strong> action. This information may <strong>be</strong> in the form <strong>of</strong>two points along the line <strong>of</strong> action or angles which orient the line <strong>of</strong>action.3. Use the dot product to compute the projection <strong>of</strong> a vector onto aspecified line and the angle <strong>be</strong>tween two vectors.4. Compute the resultant <strong>of</strong> two or more <strong>force</strong>s concurrent at a point.Moments<strong>The</strong> tendency <strong>of</strong> a <strong>force</strong> to rotate a body about an axis is descri<strong>be</strong>d<strong>by</strong> a moment (or torque), which is a vector quantity. We have seen thatfinding the moment <strong>of</strong> a <strong>force</strong> is <strong>of</strong>ten facilitated <strong>by</strong> combining the moments<strong>of</strong> the components <strong>of</strong> the <strong>force</strong>. When working with moment vectorsyou should <strong>be</strong> able to:1. Determine a moment <strong>by</strong> using the moment-arm rule.2. Use the vector cross product to compute a moment vector in terms<strong>of</strong> a <strong>force</strong> vector and a position vector locating the line <strong>of</strong> action <strong>of</strong>the <strong>force</strong>.3. Utilize Varignon’s theorem to simplify the calculation <strong>of</strong> moments,in both scalar and vector forms.4. Use the triple scalar product to compute the moment <strong>of</strong> a <strong>force</strong> vectorabout a given axis through a given point.CouplesA couple is the combined moment <strong>of</strong> two equal, opposite, and noncollinear<strong>force</strong>s. <strong>The</strong> unique effect <strong>of</strong> a couple is to produce a pure twistor rotation regardless <strong>of</strong> where the <strong>force</strong>s are located. <strong>The</strong> couple isuseful in replacing a <strong>force</strong> acting at a point <strong>by</strong> a <strong>force</strong>–couple system at


c02.qxd 10/29/07 1:38 PM Page 100100 Chapter 2 Force Systemsa different point. To solve problems involving couples you should <strong>be</strong>able to:1. Compute the moment <strong>of</strong> a couple, given the couple <strong>force</strong>s and eithertheir separation distance or any position vectors locating their lines<strong>of</strong> action.2. Replace a given <strong>force</strong> <strong>by</strong> an equivalent <strong>force</strong>–couple system, and viceversa.ResultantsWe can reduce an arbitrary system <strong>of</strong> <strong>force</strong>s and couples to a singleresultant <strong>force</strong> applied at an arbitrary point, and a corresponding resultantcouple. We can further combine this resultant <strong>force</strong> and couple intoa wrench to give a single resultant <strong>force</strong> along a unique line <strong>of</strong> action,together with a parallel couple vector. To solve problems involving resultantsyou should <strong>be</strong> able to:1. Compute the magnitude, direction, and line <strong>of</strong> action <strong>of</strong> the resultant<strong>of</strong> a system <strong>of</strong> coplanar <strong>force</strong>s if that resultant is a <strong>force</strong>; otherwise,compute the moment <strong>of</strong> the resultant couple.2. Apply the principle <strong>of</strong> moments to simplify the calculation <strong>of</strong> themoment <strong>of</strong> a system <strong>of</strong> coplanar <strong>force</strong>s about a given point.3. Replace a given general <strong>force</strong> system <strong>by</strong> a wrench along a specificline <strong>of</strong> action.EquilibriumYou will use the preceding concepts and methods when you studyequilibrium in the following chapters. Let us summarize the concept <strong>of</strong>equilibrium:1. When the resultant <strong>force</strong> on a body is zero (ΣF 0), the body is intranslational equilibrium. This means that its center <strong>of</strong> mass is eitherat rest or moving in a straight line with constant velocity.2. In addition, if the resultant couple is zero (ΣM 0), the body is inrotational equilibrium, either having no rotational motion or rotatingwith a constant angular velocity.3. When both resultants are zero, the body is in complete equilibrium.


c02.qxd 10/29/07 1:38 PM Page 101Article 2/10 Review Problems 101REVIEW PROBLEMS2/161 A die is <strong>be</strong>ing used to cut threads on a rod. If 60-N<strong>force</strong>s are applied as shown, determine the magnitudeF <strong>of</strong> the equal <strong>force</strong>s exerted on the 6-mm rod<strong>by</strong> each <strong>of</strong> the four cutting surfaces so that their externaleffect on the rod is equivalent to that <strong>of</strong> thetwo 60-N <strong>force</strong>s.Ans. F 1200 N6 mm2/163 <strong>The</strong> control lever is subjected to a clockwise couple<strong>of</strong> 80 N m exerted <strong>by</strong> its shaft at A and is designedto operate with a 200-N pull as shown. If the resultant<strong>of</strong> the couple and the <strong>force</strong> passes through A,determine the proper dimension x <strong>of</strong> the lever.Ans. x 266 mm200 N150 mmx20°60 N120mmProblem 2/1612/162 Using the principles <strong>of</strong> equilibrium to <strong>be</strong> developedin Chapter 3, you will soon <strong>be</strong> able to verify thatthe tension in cable AB is 85.8% <strong>of</strong> the weight <strong>of</strong>the cylinder <strong>of</strong> mass m, while the tension in cableAC is 55.5% <strong>of</strong> the suspended weight. Write eachtension <strong>force</strong> acting on point A as a vector if themass m is 60 kg.B120mm0.8 m 2 mC60 NProblem 2/1632/164 <strong>The</strong> blades <strong>of</strong> the portable fan generate a 4-Nthrust T as shown. Compute the moment M O <strong>of</strong>this <strong>force</strong> about the rear support point O. For comparison,determine the moment about O due to theweight <strong>of</strong> the motor–fan unit AB, whose weight <strong>of</strong>40 N acts at G.A15°100 mmGBAAir flowT = 4 N1.2 my250 mm50mmAmProblem 2/162xO50mm200 mmProblem 2/164


c02.qxd 10/29/07 1:38 PM Page 102102 Chapter 2 Force Systems2/165 For the angular position 60 <strong>of</strong> the crank OA,the gas pressure on the piston induces a compressive<strong>force</strong> P in the connecting rod along its centerlineAB. If this <strong>force</strong> produces a moment <strong>of</strong> 720N m about the crank axis O, calculate P.Ans. P 9.18 kN2/167 Represent the resultant <strong>of</strong> the three <strong>force</strong>s and couple<strong>by</strong> a <strong>force</strong>–couple system located at point A.Ans. R 4.75 kN, M A 21.1 kN m CCW3 kN1.5 m3.5 m2.5 mB10 kN·mA5 kN160°2 m4 kNOA = 125 mmAB = 300 mmProblem 2/167AθO2/168 Reduce the given loading system to a <strong>force</strong>–couplesystem at point A. <strong>The</strong>n determine the distance xto the right <strong>of</strong> point A at which the resultant <strong>of</strong> thethree <strong>force</strong>s acts.800 N720 NProblem 2/1652/166 Calculate the moment M O <strong>of</strong> the 250-N <strong>force</strong> aboutthe base point O <strong>of</strong> the robot.A200 mm450 mm500 mm20°1200 NA60°400 mmC250 NProblem 2/168500 mmBO300 mmProblem 2/166


c02.qxd 10/29/07 1:38 PM Page 103Article 2/10 Review Problems 1032/169 <strong>The</strong> directions <strong>of</strong> rotation <strong>of</strong> the input shaft A andoutput shaft B <strong>of</strong> the worm-gear reducer are indicated<strong>by</strong> the curved dashed arrows. An inputtorque (couple) <strong>of</strong> 80 N m is applied to shaft A inthe direction <strong>of</strong> rotation. <strong>The</strong> output shaft B suppliesa torque <strong>of</strong> 320 N m to the machine which itdrives (not shown). <strong>The</strong> shaft <strong>of</strong> the driven machineexerts an equal and opposite reacting torqueon the output shaft <strong>of</strong> the reducer. Determine theresultant M <strong>of</strong> the two couples which act on the reducerunit and calculate the direction cosine <strong>of</strong> Mwith respect to the x-axis.Ans. M 320i 80j N mcos x 0.970z2/171 When the pole OA is in the position shown, the tensionin cable AB is 3 kN. (a) Write the tension <strong>force</strong>exerted on the small collar at point A as a vectorusing the coordinates shown. (b) Determine themoment <strong>of</strong> this <strong>force</strong> about point O and state themoments about the x-, y-, and z-axes. (c) Determinethe projection <strong>of</strong> this tension <strong>force</strong> onto line AO.Ans. (a) T AB 2.05i 1.432j 1.663k kN(b) M O 7.63i 10.90j kN m(c) T AO 2.69 kNOA = 10 mOB = 8 mOG = 6 mzGAy320 N·mB60°B80 N·mO35°150 mmxxAyProblem 2/171Problem 2/1692/170 Determine the moment <strong>of</strong> the <strong>force</strong> P about point A.x2/172 A <strong>force</strong> F acts along the line AB inside the rightcircular cylindrical shell as shown. <strong>The</strong> quantitiesr, h, , and F are known. Using the x-, y-, andz-coordinates shown, express F as a vector.yAhAbbbzB34PyzOFBθrxProblem 2/170Problem 2/172


c02.qxd 10/29/07 1:38 PM Page 104104 Chapter 2 Force Systems2/173 Three couples are formed <strong>by</strong> the three pairs <strong>of</strong>equal and opposite <strong>force</strong>s. Determine the resultantM <strong>of</strong> the three couples.Ans. M 20i 6.77j 37.2k N m2/175 <strong>The</strong> combined action <strong>of</strong> the three <strong>force</strong>s on the baseat O may <strong>be</strong> obtained <strong>by</strong> establishing their resultantthrough O. Determine the magnitudes <strong>of</strong> Rand the accompanying couple M.Ans. R 10.93 kN, M 38.9 kN m180mm100 N80 N100 N 20°z4 kNx100mm200mm20°80 N45°30° y120 N2.8 m1.2 m1.2 m1.2 m45°120 NyO45°6 kNzx5 kNProblem 2/1732/174 During a drilling operation, the small robotic deviceis subjected to an 800-N <strong>force</strong> at point C asshown. Replace this <strong>force</strong> <strong>by</strong> an equivalent <strong>force</strong>–couple system at point O.z300 mm20°Problem 2/175*Computer-Oriented Problems*2/176 Four <strong>force</strong>s are exerted on the eyebolt as shown. Ifthe net effect on the bolt is a direct pull <strong>of</strong> 1200 Nin the y-direction, determine the necessary values<strong>of</strong> T and .AB480 NyT250 mmyCθ30°800 NOx30°F = 800 N720 N30°xProblem 2/174Problem 2/176


c02.qxd 10/29/07 1:38 PM Page 105Article 2/10 Review Problems 105*2/177 <strong>The</strong> <strong>force</strong> F is directed from A toward D and D isallowed to move from B to C as measured <strong>by</strong> thevariable s. Consider the projection <strong>of</strong> F onto lineEF as a function <strong>of</strong> s. In particular, determine andplot the fraction n <strong>of</strong> the magnitude F which isprojected as a function <strong>of</strong> s/d. Note that s/d variesfrom 0 to 22.A2dFCAns. n sDEB2d2 s d 15 s d 2 5 22 s ddF*2/179 With the cylindrical part P <strong>of</strong> weight 1500 N in itsgrip, the robotic arm pivots about O through therange 45 45 with the angle at A lockedat 120. Determine and plot (as a function <strong>of</strong> ) themoment at O due to the combined effects <strong>of</strong> theweight <strong>of</strong> part P, the 600-N weight <strong>of</strong> mem<strong>be</strong>r OA(mass center at G 1 ), and the 250-N weight <strong>of</strong>mem<strong>be</strong>r AB (mass center at G 2 ). <strong>The</strong> end grip isincluded as a part <strong>of</strong> mem<strong>be</strong>r AB. <strong>The</strong> lengths L 1and L 2 are 900 mm and 600 mm, respectively.What is the maximum value <strong>of</strong> M O and at whatvalue <strong>of</strong> does this maximum occur?Ans. M O 1845 cos 975 cos(60 ) N m(M O ) max 2480 N m at 19.90OL 1—–2L 1—–2AL 2—– 2120° G 2G 1 BθL 2—– 2PProblem 2/177*2/178 <strong>The</strong> throttle-control lever OA rotates in the range0 90. An internal torsional return springexerts a restoring moment about O given <strong>by</strong> M K( /4), where K 500 N mm/rad and is inradians. Determine and plot as a function <strong>of</strong> thetension T required to make the net moment aboutO zero. Use the two values d 60 mm and d 160 mm and comment on the relative design merits.<strong>The</strong> effects <strong>of</strong> the radius <strong>of</strong> the pulley at B arenegligible.Problem 2/179θAB40 mm45° 40 mmOTMdProblem 2/178


c02.qxd 10/29/07 1:38 PM Page 106106 Chapter 2 Force Systems*2/180 A motor attached to the shaft at O causes the armOA to rotate over the range 0 180. <strong>The</strong> unstretchedlength <strong>of</strong> the spring is 0.65 m, and it cansupport both tension and compression. If the netmoment about O <strong>must</strong> <strong>be</strong> zero, determine and plotthe required motor torque M as a function <strong>of</strong> .y*2/182 <strong>The</strong> tension T in cable AB is maintained at aconstant value <strong>of</strong> 120 N. Determine the momentM O <strong>of</strong> this tension about point O over the range0 90. Plot the x-, y-, and z-components <strong>of</strong>M O as functions <strong>of</strong> .TBM0.5 mA′′ θA′Ok = 600 N/m0.3 mAx800 mmy400 mmxBOAProblem 2/180*2/181 A flagpole with attached light triangular frame isshown here for an arbitrary position during itsraising. <strong>The</strong> 75-N tension in the erecting cable remainsconstant. Determine and plot the momentabout the pivot O <strong>of</strong> the 75-N <strong>force</strong> for the range0 90. Determine the maximum value <strong>of</strong>this moment and the elevation angle at which itoccurs; comment on the physical significance <strong>of</strong>the latter. <strong>The</strong> effects <strong>of</strong> the diameter <strong>of</strong> the drumat D may <strong>be</strong> neglected.1350 sin ( 60)Ans. M O 45 36 cos ( 60) k N m(M O ) max 225 N m at 60θ800 mmProblem 2/182A′zCA3 mBD3 mO3 mθ6 mProblem 2/181


c02.qxd 10/29/07 1:38 PM Page 107Article 2/10 Review Problems 107*2/183 <strong>The</strong> arm AB rotates in the range 0 180,and the spring is unstretched when 90. Determineas a function <strong>of</strong> the moment about O <strong>of</strong>the spring <strong>force</strong> as applied at B. Plot the threescalar components <strong>of</strong> M O , and state the maximumabsolute value <strong>of</strong> each component.1.5 0.5 sin 1Ans. M O [674i 337 cos k] N m1.5 0.5 sin (M Ox) max = 123.7 N m at 0, 180(M Oz) max = 61.8 N m at 0, 180*2/184 <strong>The</strong> rectangular plate is tilted about its loweredge <strong>by</strong> a cable tensioned at a constant 600 N. Determineand plot the moment <strong>of</strong> this tensionabout the lower edge AB <strong>of</strong> the plate for the range0 90.z5 mT = 600 NyCz3 mA360 mmO5 mθA4 mBBxProblem 2/184k = 2.6 kN/m720 mmxθODyC360 mmProblem 2/183

Hooray! Your file is uploaded and ready to be published.

Saved successfully!

Ooh no, something went wrong!